Download as pdf or txt
Download as pdf or txt
You are on page 1of 50

CNS Tumours

CNS
Tumours
By: Dr. Abdullah Nouh

1
CNS Tumours
EDiR Notes
• Medulloblastoma is hyperdense & enhancing.
• Lymphoma is isodense/hyperdense with homogenous enhancement and no calcification.
• Hamartoma of tuber cinereum is seen in children < 2yr, which leads to precocious puberty. It is attached to
the mamillary bodies with a thin stalk.
• Craniopharyngiomas cause growth failure & visual field defects.
• Kallman syndrome causes hypogonadism in later age.
• Pituitary adenomas are seen mostly in girls (9-13yrs).
• Colloid cysts are hyperdense on CT & hyperintense on T1 & T2WI.
• Epidermoid is non-enhancing.
• Schwannoma is hyperintense on T2 but meningioma is iso – hypointense.
• Schwannoma forms an acute angle with the petrous bone but meningioma forms an abtuse angle.
• Germinoma (hyperdense & enhancing) is present in pineal & suprasellar region & shows central calcification.
• Pineal germinoma (pinealoma) causes precocious puberty.
• Low-grade astrocytomas do not enhance & do not show any surrounding edema.
• Cyst wall of hemangioblastoma does not usually enhance.
• Juvenile pilocytic astrocytomas are not associated with feeding vessels (hemangioblastoma shows).
• Chiasmal glima are associated with NF1 & causes enlargement and enhancement of chiasm with extension
into the optic tract.
• Irregular peripheral enhancement with central necrosis is seen in GBM causing edema & mass effect.
• Lymphoma is hyperdense on CT and shows resolution by steroids or radiotherapy.
• Meningioma causes adjacent hyperostosis of bones.
• Germinomas are rounded lesions arising from pineal & suprasellar region. In pineal, they are mostly seen
adjacent to tectal plate & causes Parinaud syndrome → Paralysis of upward gaze due to compression of
tectum.
• They are usually not calcified & cause precocious puberty.
• Intraventricular meningiomas (2-5%) are rare but if present they are in trigone location & in middle age
females. (Hyperdense on CT).
• Pituitary macroadeoma are slightly hyperdense containing several lucent foci with bone erosions, sella
expansion, compression of optic chiasm and may cause III, IV& VI cramial nerve palsy due to extension into
cavernous sinus.
• Giant cell astrocytoma occurs at foramen of Monro & shows calcification & enhancement.
• Hemangioblastomas are large cystic masses with an enhancing mural nodule. Oedema may be absent or
extensive but no calcification is seen.
• Pilocytic astrocytomas (larger > 5cm) are similar to hemangioblastomas but show calcification, occur in
children & young adults. It is not associated with polycythaemia or draining vessel
• Rathke’s cysts are anterior & cephalic to Tornwaldt’s cyst.
• GBM rarely calcifies.
• Ependymomas demonstrate fluid levels due to internal haemorrhage.
• Central neurocytoma is intraventicular tumour causing hydrocephalus. The lesion is attached to septum
pellucidum with foci of calcification.
• Chordoma shows ‘soap bubble’ appearance.
• Ependymoma is lobulated mass is 4th ventricle, extending via the foramen of magendie into cistern magna.
• Neurinomas are nerve sheath tumours.
• Solid hypodense mass in posterior aspect of 3rd ventricle is pineal germinoma.
• Cerebral hamartomas are seen in BG & Pons & appear hyperintense on T2WI.
• Melanoma & mucinous adenocarcinoma mets are T1 hyperintense & T2 hypointense.
• Prolactinomas are most common functioning pituitary microadenomas arising from the anterior lobe of
pituitary & show poor enhancement.
• Medulloblastoma is a hyperdense mass with patchy enhancement.
• Lymphoma in AIDS patient occur in periventricular location (hyperdense) with subependymal spread &
crossing of the corpus callosum with frequent ring enhancement (homogenous enhancement of lymphoma is
seen in immunocompetent patients).
• Lymphoma is usually hypointense on FLAIR.
• Gliomatosis cerebri involves two or more lobes with contiguous involvement. Cerebral architecture is
preserved with T2 hyperintensity in white matter & deep grey nuclei with enlargement of cerebral structures.
It is bilateral & symmetrical.

2
CNS Tumours
• Periventricular & subependymal location is characteristic of lymphoma in AIDS patient.
• CNS lymphoma shows avid uptake on thallium scan but toxoplasmosis does not.
• Lyphoma shows increased vascularity but toxoplasmosis is hypovascular on MR perfusion studies.
• Rathke’s cleft cysts do not calcify.
• Glioma has greatest incidence among all cranial neoplasms.
• Choroid plexus papilloma, acoustic neuroma, hemangiopericytoma, meningioma, chordoma & epidermoid
are extra axial tumours.
• Ependymomas > Astrocytomas > Choroid plexus papillomas are intraventricular tumours.
• Widened subarachnoid cistern is seen in extra- axial lesion/tumour.
• Arachnoid cyst has smooth margins but epidermoid is usually hyperdense to CSF, encases vessels & deviates
from CSF.
• Colloid cyst can occasionally widen septum pellucidum.
• Dural meningeal carcinomatosis shows short curvilinear discontinuous thin sections of enhancement than
leptomeningeal carcinomatosis.
• Squamous cell & adenocarcinoma of lung causes cystic mets to brain.
• Oligodendroglioma most commonly affects frontal lobe but corpus callosum can also be affected as butterfly
glioma.
• Pituitary microadenomas are typically hypointense compared on normal gland on T1WI. Contrast
enhancement is seen on delayed (>20min) images.
• Intra axial lesion causes expansion of cortex of brain.
• CP angle meningioma makes obtuse angle with the petrous bone but schwannoma makes acute angle.
• Meningioma causes extension into the internal auditory canal but does not expand it.
• CP schwannoma causes extension & expansion of IAC & expansion with flaring of porus acousticus.
• A low attenuation lesion in brain in an acute hemiparesis patient of earlier operated lung cancer should get a
contrast CT done to rule out mets vs infarct.
• Ependymomas most commonly arise in the floor of 4th ventricle. They show calcification & extend through
& widen the foramen of Luschka & Magendie.
• Calcification is not commonly seen (10-20%) in medulloblastoma. Mostly are hyperdense solid tumours.
• GBM commonly spreads along white matter tracts & across the corpus callosum to involve both frontal lobes
(butterfly glioma).
• Oligodendrogliomas may erode the inner table of skull. Surrounding edema is usually minimal.
• Gangliogliomas show calcification (33%) & cyst formation (50%) & occur in children & young adults (< 30
yrs) with predilection for temporal lobes.
• Astrocytomas can also show calcification but does not cause calvarial erosion.
• Ganglioglioma is most common tumour of temporal lobe epilepry.
• DNET are associated with partial complex seizures & occur before 20yrs. There is soap bubble, multicystic
lesion with remodeling of calvarium.
• Parinaud syndrome = Dorsal midbrain syndrome. It occurs due to injury or compression of midbrain
especially superior colliculus with pineal germinoma.
• Meningioma is most common radiation induced CNS tumour. Usually they are multiple.
• Colloid cyst causes positional headaches.
• Suprasellar meningiomas are not cystic like craniopharyngiomas.
• Epidermoid contains ectodermal component but dermoid contains ectodermal & mesodermal contents.
• CNS teratomas occur in pineal & suprasellar regions.
• Corpus callosal lipoma is associated with its agenesis, encephalocele & cutaneous frontal lipomas.
• Interhemispheric arachnoid cysts can also occur with agenesis of corpus callosum.
• Opsoclonus – myoclonus syndrome is characterized by association with breast & Squamous cell lung Ca &
neuroblastoma (children). Therefore, in children, MIBG scan should be done since MRI brain is negative.
• Central neurocytomas are benign tumours of lateral & third ventricles, attached to the septum pellucidum &
contains Ca (69%) & cystic spaces. Subependymoma can have similar picture but occur in age > 40 yrs.
• Choroid plexus papilloma occurs < 5yrs of age.
• Pinealoma can cause obstruction at the level of aqueduct.
• Pinealoma is located between the splenium of Corpus callosum & tectal plate.
• Meningeal seedings are very common in pineal germinomas & pineoblastomas.
• Avid enhancement is seen in pineal germinoma. Central calcification is seen in germinoma but peripheral
calcification is seen in pineoblastoma. (Both can cause parinaud syndrome).
• Pleomorphic xanthoastrocytomas show cysts but no calcification or hemorrhage & are seen in temporal lobe.

3
CNS Tumours
• MERRF causes hyperintense FLAIR pathologies of BG, caudate nuclei, & watershed ischemia.
• Lymphoma in HIV can cause encasement of the ventricle, which is not seen with toxoplasma.
• Hemangiopericytoma is an extra axial tumour having broad dural base & dural tail, shows heterogenous
enhancement with no e/o calcification, abutting the sphenoid sinus in anterior cranial fossa with displacement
& effacement of third ventricle causing dilated lateral ventricles.
• Tentorial or CP angle meningiomas are supplied by Bernasconi – Casanari artery, a branch of ICA.
• Parafalcine meningiomas or meningiomas around cerebral convexity or sphenoidal meningiomas are fed by
ECA.
Pilocytic astrocytoma Medulloblastoma
No seeding Subarachnoid seeding
Arise peripherally & displace 4th vent Arise centrally from vermis
Cystic or solid Solid (not differentiating)
• Malignant meningioma involves intraparenchymal tissue of brain as well.
• Calvarial scalloping can be seen with arachnoid cyst but not with subdural hygroma.
• Pilocytic astrocytoma does not show surrounding vasogenic edema.
• NF1, optic glioma, pineal germinoma & pilocytic astrocytoma are associated with precocious puberty.
• Elevated cholnic, reduced NAA & Choline / Creatine ratio of 2 indicates high grade tumour.
• Epidermoids are iso-hyperintense on FLAIR but arachnoid cysts are hypointense.
• Hamartoma of tuber cinerium causes gelastic seizures.
• Hamartomas of tuber cinereum are developmental malformations which do not change in size, shape & signal
intensity on follow up. No enhancement or calcification is seen.
• Rathke’s cleft cysts are hyperintense on T1 & hypointense on T2WI. Located midline between anterior &
posterior lobes of pituitary & have a kidney shape on axial images. They do not enhance.
• Acute hemorrhagic pituitary adenoma shows fluid levels.
• Autoimmune hypophysitis shows loss of posterior pituitary bright spot, homogenous enhancement, &
enlarged stalk.
• Normal posterior pituitary is bright on T1WI due to rich content of vasopressin neurosecretary granules.
• Schwannoma can cause erosion of adjacent porus acousticus.
• Elevated lactate on MR spectroscopy is a sign of high grade lesions.
• 1.75 is the threshold of rCBV value in MR perfusion between low grade & high grade tumours.
• Craniopharyngiomas contain highly proteinaceous fluid, cholesterol & blood products causing high signal on
T1, T2 & FLAIR images. There can be mild to minimal peripheral enhancement & fluid-fluid levels.
• Craniopharyngiomas have bimodal age distribution (5-10 yrs & 50-60 yrs). High signal on T1 & T2 are due
to cholesterol crystals in the cyst. Solid portions enhance.
• Diffuse brainstem glioma shows a non enhancing mass causing expansion of pons with engulfing of basilar
artery.
• Scalp dermoid shows a central lucent area with sclerotic margins.
• Pilocytic astrocytomas are associated with NF1 & they may show calcification (not seen in
hemangioblastoma).
• Epidermoid and dermoid cysts both cause scalloping and sclerosis with destruction of adjacent bone.
Epidermoid calcify rarely but dermoid cysts calcify commonly.
• Intraventricular meningiomas are seen above 20 years of age.
• Choroid plexus papilloma shows a mildly hyperdense homogeneous mass at the trigone of lateral ventricle
with intense homogeneous enhancement. It is hyperintense on T1WI & slightly hypointense on T2WI.
• Hypothalamic hamartoma extends into suprasellar cistern or interpeduncular cistern.
• LCH in skull gives ovoid lesion with bevelled edge without marginal sclerosis, giving a punched out
appearance. There may be an associated soft tissue mass overlying the lytic process which is often palpable.
It may affect the long bones in children commonly (diaphysis).
• Medulloblastoma are hyperdense midline vermian mass at the roof of 4th ventricle causing hydrocephalus.
• PNET (Medulloblastoma & pineoblastoma) shows calcification, haemorrhage & necrosis (like
craniopharyngioma) but they are mostly seen is posterior fossa.
• Craniopharyngioma can cause visual problems, diabetes insipidus & growth problems duo to pressure/mass
effect on optic chiasm, pituitary & hypothalamus.
• DNET arises from cortex in temporal > frontal region, appears as hypodense mass with remodelling/thinning
of underlying inner table. Intratumoral cysts giving “bubbly” appearance is seen. Minimal enhancement or
mass effect is seen.
• Choroid plexus tumour occurs adjacent to the trigone of ventricles, appearing hyperdense, enhancing &
causing hydrocephalus.

4
CNS Tumours
• Teratomas may enhance but dermoid cysts do not. Dermoids are usually extra-axial & teratomas are usually
around pineal region, floor of 3rd ventricle, posterior fossa & spine.
• Calcifications can be seen in both dermoids & epidermal cysts. Focal low attenuation on CT is seen in
dermoid & diffusely low attenuation in epidermoid.
Succeeding
1.Which of the following features favour Rathke’s cleft cyst rather than craniopharyngioma?
A. Cystic element on MR
B. Involvement of suprasellar and sellar regions
C. Enhancement of the wall
D. Absence of calcification
E. High signal intensity on T1

1. D
Rathkes cleft cysts do not calcify. They affect women to men in a 2:1 ratio and adults from 40-60 years of age.
They cause variable MR appearances depending on protein content of cyst. They can rarely show enhancement.
9. Which brain tumour has the greatest incidence across all age groups?
A. Meningioma
B. Metastases
C. Pituitary adenoma
D. Haemangioblastoma
E. Glioma

9. E
Gliomas consist of astrocytomas, oligodendrogliomas, paragangliomas, ganglogliomas and medulloblastomas.
10. Which of the following is an extra-axial posterior fossa tumour in adults?
A. Metastasis
B. Haemangioblastoma
C. Choroid plexus papilloma
D. Lymphoma
E. Glioma

10. C
Other extra-axial posterior fossa masses include acoustic neuroma, meningioma, chordoma and epidermoid.
(Ped) 13. Which is the most common site of metastatic spread in medulloblastoma?
A. Axial skeleton
B. Lymph nodes
C. Lung
D. Subarachnoid space
E. Liver

13. D
Subarachnoid space is the most common, with drop metastases occurring in 40%.
14. Which of the following features describes an intra-axial mass?
A. Cortex displaced towards bone
B. Mass contiguous with dura
C. Buckling of grey and white matter
D. Widened subarachnoid cistern
E. Dural feeding arteries

14. A
B-E are extra-axial features.
15. Which of the following favours an arachnoid rather than an epidermoid cyst?
A. Hyperdense to CSF on CT
B. Encased vessels
C. Deviates from CSF on proton density
D. Restricted diffusion
E. Smooth margin

5
CNS Tumours
15. E
A and C-E are features of epidermoid cyst. A and C demonstrate CSF-like density and have smooth margins.
18. Which of the following best describes imaging changes in a colloid cyst?
A. Typically hypodense on non-contrast CT
B. Appears high SI on T1
C. Appears low SI on T2
D. Commonly widens septum pellucidi
E. Most commonly causes symmetrical enlargement of lateral ventricles

18. B
Protein content/paramagnetic effect of magnesium Mg2+/ calcium Ca2+/, Iron Fe, in a cyst cause increased T1
and T2 SI. Colloid cysts appear iso/hyperdense on NCCT. They can occasionally widen septum pellucidum and
cause asymmetrical enlargement of the lateral ventricles.
19. Which is the most common location for epidermoid in the Central Nervous System (CNS)?
A. Cerebellar pontine angle (CPA)
B. Suprasellar region
C. Perimesencephalic cisterns
D. Ventricles
E. Skull vault

19. A Located in CPA in 40%, accounting for 5% of CPA tumours.


21. Which of the following favours dural meningeal carcinomatosis rather than leptomeningeal cacinomatosis?
A. Positive cytology
B. Short discontinuous thin sections of enhancement
C. Thin area of subarachnoid enhancement following convulsions of gyri
D. Discrete leptomeningeal nodules
E. Invasion of underlying brain with mass effect and oedema

21. B
Dural meningeal carcinomatosis is rarely associated with positive cytology and involves localised or diffuse
curvilinear enhancement underneath inner table in expected position ofdura.
22. Which is the cause of a cystic rather than a haemorrhagic cause of brain metastases?
A. Malignant melanoma
B. Choriocarcinoma
C. Renal cell carcinoma
D. Thyroid carcinoma
E. Adenocarcinoma of the lung

22. E
Squamous cell lung cancer and adenocarcinoma of the lung cause cystic metastasis to the brain. Answers B-E
are causes of haemorrhagic metastases.
23. Which is the most common location of oligodendroglioma?
A. Temporal lobe
B. Parietal lobe
C. Occipital lobe
D. Frontal lobe
E. Cerebellum

23. D
Most commonly involve cortical & subcortical white matter, occasionally through CC as butterfly glioma.
33. A 2cm mass is seen on MR at the left CPA with uniform enhancement and high SI on T2 and dural tail. What
is thediagnosis?
A. Vestibular schwannoma
B. Epidermoid
C. Metastatic deposit
D. Meningioma
E. Glomus tumour

6
CNS Tumours
33. D
Broad based attachment to petrous bone. More homogenousSI and less bright T2. Uniform enhancement
distinguishesfrom vestibular schwannoma.
Get through
6) Which of the following conditions will typically demonstrate unrestricted MR DWI and ADC map?
a. epidermoid cyst
b. acute infarction
c. cerebral abscess
d. glioblastoma multiforme
e. viral encephalitis

6) d. __
Diffusion-weighted MRI provides image contrast which is different from that provided by conventional MR
techniques. The sequence enables the measurement of net macroscopic water movement, which is anisotropic
(varies in different directions) particularly in white matter. Restricted diffusion is seen as high signal on DWI
(which is a T2W image with signal degraded by diffusion) and low signal on the ADC map. Restricted diffusion
occurs in tissue that does not allow free movement of water molecules, such as areas of infection due to the high
viscosity and cellularity of pus. Similarly, epidermoid cysts are very cellular and so also show restricted diffusion,
a feature that helps distinguish them from arachnoid cysts, which are fluid structures. In stroke, restriction in
water diffusion occurs within minutes after the onset of ischaemia. The basis of this change is not completely
clear but is thought to be related to the cytotoxic oedema seen in ischaemic cells due to the impairment of the
Na+/K+ ATPase pumps (which are very energy dependent), leading to loss of ionic gradients and a net
translocation of water from the extracellular to the intracellular compartment, where water mobility is relatively
more restricted.
13) What are the typical appearances of a pituitary microadenoma on early, contrast-enhanced T1W MR images?
a. focus of hypointensity within normal enhancing gland
b. focus of enhancement within normal, non-enhancing gland
c. lesion and normal gland enhance similarly
d. hyperenhancing focus within normal, mildly enhancing gland
e. not usually visualized on this sequence

13) a. ___
Pituitary microadenomas are typically hypointense compared with the normal gland on unenhanced T1W images,
and the diagnosis can usually be made without contrast. Following contrast, the microadenoma does not initially
enhance, and maximal contrast between enhancing normal gland and pituitary tumour is seen on dynamic images
obtained within the first minute. Contrast enhancement may therefore be useful inidentifying lesions that are not
obviously hypointense on the unenhanced images. However, contrast enhancement of the tumour relative to the
normal gland may be seen on delayed (.20 min) images.

(Ped) 15 A 7 year old girl presents with lethargy, headaches and vomiting. CT shows a hyperdense lesion in the
region of the posterior fossa. MR imaging confirms a midline vermian mass which abuts the roof of the 4th
ventricle, displacing the brainstem anteriorly. The mass is hypointense on T2 and enhances homogeneously on
T1 following i. v. contrast. What is the most likely diagnosis?
(a) Brainstem glioma
(b) Ependymoma
(c) Haemangioblastoma
(d) Medulloblastoma
(e) Pilocytic astrocytoma

15 (d)
All are examples of posterior fossa masses in children; other causes include meningioma, and epidermoid or
dermoid cysts. These features are typical for medulloblastoma (the differential diagnosis is an atypical teratoid /
rhabdoid tumour). Pilocytic astrocytomas are cystic with an enhancing peripheral nodule, ependymomas arise
from 4th ventricle floor and are hypodense on CT, haemangioblastomas enhance avidly.

7
CNS Tumours
17) A mass is seen peripherally in the middle cranial fossa on MR of the brain. Which of the following imaging
features favours an intra-axial rather than an extra-axial location?
a. buckling of the grey–white matter interface
b. expansion of the cortex of the brain
c. expansion of the subarachnoid space
d. medial displacement of pial blood vessels by the mass
e. the mass has a dural base

17) b. ___
Once the presence of a mass has been established, the radiologist must determine whether the mass is intra-axial
(arising within the brain parenchyma) or extra-axial (arising outside the brain substance) in order to formulate an
appropriate differential diagnosis. An extra-axial mass characteristically causes buckling of the grey–white matter
interface, expansion of the subarachnoid space at its borders, and medial displacement of the vessels in the
subarachnoid space. A dural base is also a feature of an extra-axial mass. Intra-axial masses characteristically
cause expansion of the cortex of the brain but no expansion of the subarachnoid space, and pial vessels may be
seen peripheral to the mass.
18) A 35-year-old woman presents with progressive deafness and tinnitus in the left ear. She undergoes MRI,
which demonstrates a 2 cm mass at the left cerebellopontine angle. Which of the following features would favour
a diagnosis of meningioma rather than vestibular schwannoma?
a. acute angle with the petrous bone
b. hyperintensity on T2W images
c. expansion of the internal auditory canal
d. presence of a dural tail
e. internal cystic degeneration and haemorrhage

18) d. ___
The most common causes of a cerebellopontine angle mass are vestibular schwannoma (also called acoustic
neuroma) (75%), meningioma (10%) and epidermoid cyst (5%). Features suggestive of a meningioma include a
dural tail (thickening of enhancing adjacent dura resembling a tail extending from the mass), adjacent hyperostosis
and an obtuse angle with the petrous bone (vestibular schwannomas make an acute angle). Distinguishing features
of schwannomas include extension into the internal auditory canal, causing expansion of the canal and flaring of
the porusacousticus (bony opening of the internal auditory canal). Meningiomas may show a small tongue of
extension into the canal but usually no expansion. Schwannomas undergo cystic degeneration and haemorrhage
more commonly than meningiomas (particularly larger lesions), and may show very high signal on T2W images,
which is unusual for a meningioma.
25) A 70-year-old man is referred for CT scan of the brain due to sudden onset of left-sided hemiparesis and
clinical diagnosis of stroke. Unenhanced CT shows a rounded area of low attenuation in the right posterior frontal
lobe with local gyriform swelling and sulcal effacement. Upon questioning, he reveals a history of lung resection
for malignancy 18 months previously. Which of the following imaging investigations would you perform next?
a. no further imaging
b. CTof the thorax
c. CTof the thorax and abdomen
d. MRI of the brain
e. contrast-enhanced CTof the brain

25) e. __
In this scenario, the low attenuation and surrounding changes most likely represent brain oedema. This may be
due to an evolving infarction or oedema around a metastatic deposit from the previous lung cancer. Differentiation
between the two will immediately affect patient treatment, as anti-platelet therapy for ischaemic stroke will
increase the risk of haemorrhage from a metastasis and therefore should be with held if a metastatic deposit is
suspected or diagnosed. The primary factor in determining whether a lesion will enhance on CTafter
administration of intravenous iodinated contrast is the integrity of the blood–brain barrier in that region of the
brain substance. A large molecule such as iodinated contrast would not be able to enter the brain unless the
integrity of the barrier were compromised. The majority of aggressive tumours, including metastases, will disrupt
this barrier, and so contrast enhancement will be seen in the solid component of these lesions. Acute infarction
will typically not show areas of enhancement.

8
CNS Tumours
19) A 64-year-old woman presents with progressive headache and confusion. On CT, she is found to have
multiple, well-defined, rounded, low-attenuation masses of varying sizes in both hemispheres at the grey–white
matter junction. The masses demonstrate intense enhancement following intravenous contrast, and there is
considerable surrounding oedema. Which of the following is the most appropriate next imaging investigation?
a. mammography
b. thyroid ultrasound scan
c. barium enema
d. renal ultrasound scan
e. chest radiograph

19) e. ___
Brain metastases are the most common intracranial tumours. Six primary tumours account for 95% of all brain
metastases. Primary bronchial carcinoma is the most common (47% of cases), though squamous cell carcinoma
rarely metastasizes to the brain. Other common primary tumours are breast carcinoma (17%), gastrointestinal
malignancy (15%), renal cell carcinoma, melanoma and choriocarcinoma. Metastases characteristically occur at
the grey–white matter junction, are multiple in 66% of cases, and typically appear as hypodense masses that
demonstrate solid or ring enhancement.
31) A 5-year-old boy undergoes CT of the brain for investigation of headaches, vomiting and ataxia. This
demonstrates a welldefined, multilobulated, isodense mass within the fourth ventricle containing areas of punctate
calcification. The mass is seen to extend out of the foramina of Luschka into the cerebellopontine angles. There
is associated hydrocephalus. What is the most likely diagnosis?
a. metastasis
b. haemangioblastoma
c. juvenile pilocytic astrocytoma
d. medulloblastoma
e. ependymoma

31) e. ____
Ependymomas most commonly arise in the floor of the fourth ventricle and are usually isodense. They have a
greater incidence of calcification than other posterior fossa paediatric tumours; it is typically punctate and seen
in 40–50% of cases. A characteristic feature of ependymomas is their propensity to extend through and widen the
foramina of Luschkaand Magendie. Juvenile pilocyticastrocytomas are the commonest paediatric infratentorial
neoplasms and typically occur in the cerebellar hemispheres. They appear cystic with an enhancing mural nodule.
Medulloblastomas tend to be homogeneous hyperdense lesions located in the vermis, and the presence of
calcification is uncommon. Metastases are the commonest infratentorial tumour to occur in adults, but are
uncommon in children. Haemangioblastomas usually occur in young adults and are classically cystic masses with
a solid mural nodule.
39) A 65-year-old, previously well man with a short history of headaches and behavioural change undergoes CT
of the brain. This demonstrates an irregular, ill defined mass in the left frontal lobe extending across the corpus
callosum to involve the right frontal lobe. The mass is of low attenuation and contains cystic areas, demonstrates
ring enhancement following intravenous contrast, and has considerable surrounding oedema. What is the most
likely diagnosis?
a. progressive multifocal leukoencephalopathy
b. glioblastoma multiforme
c. lymphoma
d. abscess
e. metastasis

39) b. ___
Glioblastoma multiforme is the most malignant form of astrocytoma. It occurs in older patients, and most
commonly affects the deep white matter of the frontal lobes. Classic appearances are of an irregular,
illdefinedhypodense mass with necrosis, haemorrhage and extensive surrounding white matter oedema. Ninety
per cent of cases show enhancement, which may be diffuse, heterogeneous or ring like. Tumourspread is directly
along white matter tracts, and commonly occurs across the corpus callosum to involve both frontal lobes (butterfly
glioma). Lymphomas also have a propensity to involve the corpus callosum but usually are slightly hyperdense
due to a high nuclearto- cytoplasmic ratio. Metastases may also involve the corpus callosumbut tend to be better
defined and would be less likely in the absence of a known primary tumour. Progressive multifocal
leukoencephalopathy may involve the corpus callosum but occurs in immunocompromised patients. Involvement
of the corpus callosum is not usually a feature of abscesses.

9
CNS Tumours
46) A 17-year-old boy presents with headache and is found to have paralysis of upward gaze (Parinaud’s
syndrome) on examination. MR scan of the brain identifies an abnormality. What is the most likely site of lesion?
a. thalamus
b. occipital lobe
c. optic chiasm
d. pineal gland
e. cerebellar vermis

46) d. ____
Parinaud’s syndrome (also known as dorsal midbrain syndrome) is characterized by supranuclear paralysis of
upward gaze. It results from injury or compression of the dorsal midbrain, in particular the superior colliculi, and
is most commonly seen in young patients with tumours of the pineal gland or midbrain, with pineal germinoma
being the most common lesion producing the syndrome. Young women with multiple sclerosis and elderly
patients with brain-stem stroke may also present with Parinaud’s syndrome.
47) A 28-year-old woman presents with a history of headaches and refractory temporal lobe epilepsy. CT of the
brain demonstrates a mixed solid–cystic, intraparenchymal mass located peripherally in the right temporal lobe,
which contains calcification and demonstrates faint enhancement following intravenous contrast. There is
minimal surrounding oedema. What is the most likely diagnosis?
a. arachnoid cyst
b. ganglioglioma
c. epidermoid
d. meningioma
e. dysembryoplastic neuroepithelial tumour

47) b. ____
Gangliogliomas are low-grade tumours with a good prognosis, generally occurring in patients under the age of
30. Typical presentation is with focal seizures, and ganglioglioma is the most common tumour seen in patients
with chronic temporal lobe epilepsy. They are usually well-circumscribed, hypo- or isodense lesions in the
temporal lobes. Calcification (30%) and cyst formation (.50%) are common features. There is usually minimal
mass effect and surrounding oedema. Meningiomas commonly calcify and have minimal surrounding oedema,
but are extra-axial, and usually demonstrate intense uniform enhancement following intravenous contrast.
Dysembryoplastic neuro epithelial tumours are commonly associated with partial complex seizures, but usually
occur before the age of 20, and characteristically appear as a soap-bubble, multicystic lesion, which may remodel
the calvarium. Epidermoids and arachnoid cysts are of CSF density, do not enhance with contrast and are extra-
axial lesions.

50) A 44-year-old man presents with a long history of headaches and more recent onset of seizures. CT of the
brain demonstrates an oval, well-defined, heterogeneous, hypodense mass containing large nodular clumps of
calcification located peripherally in the right frontal lobe. The mass extends to the cortical margin, and there is
erosion of the inner table of the skull. There is minimal surrounding vasogenic oedema. What is the most likely
diagnosis?
a. meningioma
b. oligodendroglioma
c. astrocytoma
d. glioblastoma multiforme
e. ganglioglioma

50) b. ____
Oligodendrogliomas are slow-growing tumours, usually presenting in adults aged 30–50 years. They occur most
commonly in the frontal lobe, and often extend to the cortex, where they may erode the inner table of the skull.
Calcification is seen in 70% of cases, typically appearing as large nodular clumps. There is usually a relative
absence of surrounding oedema. Astrocytomas also usually appear as hypodense calcified lesions with little
surrounding oedema, but calvarial erosion is not usually a feature. Glioblastoma multiforme usually has
considerable surrounding oedema and rarely calcifies. Gangliogliomas show calcification in a third of cases but
tend to occur in children and young adults, and have a predilection for the temporal lobes.

10
CNS Tumours
57) A 46-year-old man presents with headaches and visual disturbance and is found to have bitemporal
hemianopia on visual field testing. CT of the brain demonstrates a multilobulated, heterogeneous, suprasellar
mass containing cystic areas and rim-like calcification. Enhancement of the solid component is observed
following intravenous contrast. What is the most likely diagnosis?
a. epidermoid
b. craniopharyngioma
c. pituitary macroadenoma
d. meningioma
e. Rathke’s cleft cyst

57) b. ___
Craniopharyngiomas are the most common suprasellar mass, predominantly occurring in the first and second
decades, but with a second peak in the fifth decade. Presenting symptoms include headache secondary to
hydrocephalus, bitemporal hemianopia (compression of the optic chiasm) and diabetes insipidus (compression of
the pituitary gland). Typical imaging features are calcification, cyst formation and enhancement that may be solid
or nodular. Meningiomas may arise in the suprasellar region and commonly demonstrate calcification but are
generally not cystic. Epidermoids may occasionally demonstrate rim calcification but rarely enhance following
intravenous contrast. Pituitary macroadenomas may undergo haemorrhage, resulting in heterogeneity that can
cause confusion with craniopharyngioma, but this typically occurs in adolescence. Calcification in
macroadenomas is infrequent. Rathke’s cleft cysts are thin-walled, benign cysts arising in the anterior sellar or
suprasellar region. They show no contrast enhancement and rarely calcify.

58) A 27-year-old, previously well man gives a history of positional headaches. He undergoes unenhanced CT of
the brain, which demonstrates a well-defined, hyperdense,spherical lesion in the anterosuperior portion of the
third ventricle, with asymmetrical enlargement of the lateral ventricles. What is the most likely diagnosis?
a. meningioma
b. colloid cyst
c. ependymoma
d. subependymal giant cell astrocytoma
e. choroid plexus papilloma

58) b. __
Colloid cysts arise from the inferior aspect of the septum pellucidum and protrude into the anterior aspect of the
third ventricle, where they may cause positional headaches and hydrocephalus due to transient obstruction at the
foramen of Monro. They usually contain mucinous fluid, desquamated cells and proteinaceous debris, making
them hyperdense on CT. On MR scan, the high protein content, as well as the paramagnetic effect of magnesium,
copper and iron in the cyst, results in high signal intensity on T1W and T2W sequences in 60% of cases.
Meningiomas and ependymomas may also appear hyperdense, but location within the third ventricle is
uncommon for both. Subependymal giant cell astrocytoma is a benign tumour occurring in the region of the
foramen of Monro, which may cause obstruction, but it is usually hypodense and is nearly always seen in
association with tuberous sclerosis. Choroid plexus papilloma predominantly occurs in children under 5 years of
age, and location in the third ventricle is unusual.

60) Which of the following is the most common radiation-induced CNS tumour?
a. ependymoma
b. oligodendroglioma
c. lymphoma
d. glioblastoma multiforme
e. meningioma

60) e. ____
Meningioma is the most common radiation-induced CNS tumour, and has been particularly associated with low-
dose radiation treatment for tinea capitis. For the diagnosis of radiation-induced meningioma to be made, the
meningioma must arise in the radiation field, appear after a latency period of years and should not have been the
primary tumour irradiated. Radiation-induced meningiomas are more frequently multiple and have higher
recurrence rates than non-radiation-induced tumours.

11
CNS Tumours
66) Following a large postpartum haemorrhage, a 25-year-old woman develops a severe headache and sudden
visual field defect. What is the most likely diagnosis?
a. intracerebral haemorrhage
b. reversible posterior leukoencephalopathy
c. subarachnoid haemorrhage
d. Sheehan’s syndrome
e. vertebral artery dissection
66) d. ___
Many of the acute neurological conditions of pregnancy occur with rising blood pressure. Sheehan’s syndrome
results from haemorrhage-induced hypotension causing pituitary infarction. Early on, this appears as an enlarged
homogeneous pituitary with low T1 signal, high T2 signal and post-contrast ring enhancement. Later, there is an
empty sella. Clinical manifestations include visual field loss, headache, ophthalmoplegia and pituitary
dysfunction (diabetes insipidus). Reversible posterior leukoencephalopathy produces cortical blindness,
headaches, confusion and seizures. Those affected are often taking immunosuppressant treatment. Imaging
features can be identical to eclampsia, peripartum cerebral angiopathy and hypertensive encephalopathy, but with
a posterior predominance. On CT, there is low attenuation change. On MRI, there is high signal on T2W/FLAIR
images. ADC maps can differentiate between likely reversible vasogenic oedema (high signal on ADC map
showing unrestricted diffusion) and cytotoxic oedema (low signal due to restricted diffusion), which is more
likely to progress to infarct. Microangiopathic haemolytic anaemias, such as thrombotic thrombocytopenic
purpura and haemolytic uraemic syndrome, give widespread ischaemia/infarction and haemorrhagic
transformation. There is no increased risk in pregnancy of vasculitis such as systemic lupus erythematosus,
Takayasu’s syndrome or Moyamoya syndrome.Arteriovenous malformation is no more likely to bleed in
pregnancy, but there is an increased risk with arterial aneurysms. Haemorrhage, sepsisand pulmonary embolism
cause hypotension that can cause watershed infarction as well as Sheehan’s syndrome.
72) A 23-year-old man presents with acute headache. Unenhanced CT of the brain demonstrates a heterogeneous
mass at the inferior cerebellar vermis. It is predominantly of fat attenuation with areas of calcification and does
not enhance following administration of intravenous contrast. Multiple droplets of fat attenuation are noted
throughout the subarachnoid space. What is the most likely diagnosis?
a. lipoma
b. arachnoid cyst
c. dermoid
d. epidermoid
e. teratoma

72) c. ____
Epidermoids and dermoids are congenital lesions resulting from inclusion of ectodermal elements during closure
of the neural tube. Both have a squamous epithelial lining and produce keratin, but dermoids contain both
ectodermal and mesodermal elements (hair follicles, sweat and sebaceous glands), while epidermoids contain
only ectodermal elements. Epidermoids are lobulated masses, usually located off the midline, which compress
adjacent structures such as cranial nerves, and tend to have appearances on CTand MR scan following that of
CSF. Dermoids are usually midline in location and cause symptoms by obstruction of CSF pathways or by rupture
and leakage of fat contents, causing chemical meningitis. They have appearances on CTand MR scan following
that of fat, and are often heterogeneous with areas of calcification and other soft-tissue components. This
heterogeneity helps to distinguish a dermoid from a lipoma. Arachnoid cysts are of CSF density, but may be
distinguished fromepidermoids on DWI, where they do not show evidence of water restriction. Teratomas are
composed of ectodermal, mesodermal and endodermal elements, and also appear heterogeneous with areas of fat,
calcification and cystic components, but these lesions occur most commonly in the pineal and suprasellarregions.

86) A 27-year-old female presents with emotional lability and headaches. MRI of the brain demonstrates a well-
defined mass in the pericallosal region. The mass is hyperintense on T1W images and demonstrates no
enhancement with intravenous gadolinium. There is associated agenesis of the corpus callosum. What is the most
likely diagnosis?
a. dermoid
b. lipoma
c. interhemispheric arachnoid cyst
d. epidermoid
e. lymphoma

12
CNS Tumours
86) b. ___
Intracranial lipomas appear as well-circumscribed masses of fat density on CTwith occasional rim calcification.
On MR scan, characteristic appearances are of hyperintensity on T1W images, with chemical shift artefact or
signal suppression on fat-saturated sequences. Approximately 30% of intracranial lipomas occur in the
pericallosal region, and there is a high incidence of associated congenital anomalies, most commonly agenesis of
the corpus callosum, but also encephalocele and cutaneous frontal lipomas. Interhemispheric arachnoid cysts may
occur in association with agenesis of the corpus callosum, but they are of CSF density and, like epidermoids,
appear hypointense on T1W images. Dermoids have signal characteristics following those of fat, but are usually
more heterogeneous and not associated with callosal anomalies. Lymphoma may involve the corpus callosum but
appears isoorhypointense on T1W images.
(Ped) 94) A 15-year-old boy presents to accident and emergency with signs of meningitis but no pyrexia. CT and
MRI show a retroclival midline cystic tumour with localized mass effect. Which feature would support the
diagnosis of a dermoid rather than an epidermoid cyst?
a. restricted diffusion on DWI
b. no enhancement with intravenous gadolinium on MRI
c. multiple septations on either modality
d. focal low attenuation on CT
e. well-defined calcifications on CT

94) d. ___
Dermoid and epidermoid cysts are ectoderm-lined congenital inclusion cysts. They may not present until early
adulthood due to slow growth (particularly epidermoids). Epidermoids contain only squamous epithelium
whereas dermoids contain hair, sebaceous and sweat glands, and squamous epithelium. Unlike teratomas, neither
is a true neoplasm. Clinical presentation is often with chemical meningitis from rupture of fatty contents.
Epidermoids usually have imaging characteristics similar to water and can be differentiated from arachnoid cysts
by restricted diffusion on DWI. Attenuation varies according to the keratin:cholesterol ratio and therefore can be
similar to fat but is usually homogeneous. The cyst wall is very thin and is often not visible in epidermoids, and
areas of calcification can infrequently be seen. Dermoids may appear more complex but are still unilocular. The
wall may be thicker and calcification is more frequent. The sebaceous lipid material in a dermoid has attenuation
and signal intensity characteristics of fat on CT (low attenuation) and MR (high signal on T1). ‘White
epidermoids’ with high signal on T1 may be seen rarely; they are due to haemorrhage or a high fat content.
However, the latter usually produces a homogeneous fat signal as opposed to dermoids, where the appearances
are more heterogeneous due to the increased complexity of contents.
99) A 21-year-old man presents with acute headache. He undergoes CTof the brain, which demonstrates a well-
circumscribed, lobulated, partially cystic, calcified mass in the frontal horn of the left lateral ventricle, attached
to the septum pellucidum. There is acute blood seen layering in the left lateral ventricle and mild hydrocephalus.
What is the most likely diagnosis?
a. choroid plexus papilloma
b. colloid cyst
c. intraventricular oligodendroglioma
d. central neurocytoma
e. subependymoma

99) d. _____
Central neurocytomas (also known as intraventricular neurocytomas)are benign tumours of the lateral and third
ventricles usually presenting in adults aged 20–40 years. They frequently calcify (69%) and contain cystic spaces.
Attachment to the septum pellucidum is a characteristic feature. Lesions appear isointense to grey matter on all
MR sequences and show mild-to-moderate contrast enhancement. Central neurocytomas were previously
frequently mistaken for intraventricular oligodendrogliomas, which have very similar imaging features but are
actually quite rare. In addition, neurocytomas undergo haemorrhage into the tumour or ventricle more frequently,
helping to distinguish the two. Colloid cysts arise within the third ventricle, and rarely calcify. Subependymomas
may arise in the lateral ventricles with an attachment to the septum pellucidum, and cyst formation and
calcification may be seen in large tumours. However, most occur in patients over 40 years of age. Choroid plexus
papillomas generally occur in children under 5 years of age.

13
CNS Tumours
100) A 2-year-old boy presents with involuntary saccadic eye movements (opsoclonus) and myoclonus of the
trunk and limbs. What are the most likely findings on MRI of the brain?
a. normal appearances
b. cystic lesion in the posterior fossa with enhancing mural nodule
c. cerebellar atrophy
d. central symmetrical lesions in the pons of high signal intensity onT2W images
e. caudally displaced brain stem and fourth ventricle with tonsillarherniation

100) a. ____
Opsoclonus–myoclonus syndrome is characterized by opsoclonus incombination with myoclonus of the trunk,
limbs or head. It may be idiopathic or occur as a paraneoplastic syndrome, when it may follow are lapsing–
remitting course. In adults, it is most commonly associated with breast and squamous cell lung carcinoma, while
in children it is associated with neuroblastoma. The syndrome usually precedes diagnosis of the underlying
malignancy, and in children should prompt investigation to identify an underlying neuroblastoma. MIBG whole-
body scintigraphy may be helpful in identifying occult disease if conventional imaging is negative. MR scan of
the brain is usually normal.
Barret1
3 A 76 year man presents with right-sided weakness. There is a low-attenuation lesion in the left basal ganglia.
An MRI is arranged for further evaluation. Which feature would suggest a high-grade neoplasm rather than an
infarct?
(a) Involvement of both cerebral cortex and juxtacortical white matter
(b) Gyriform enhancement
(c) Low signal on an apparent diffusion coefficient image
(d) Elevated choline on magnetic resonance spectroscopy
(e) Cytotoxic oedema

3 (d)
MR spectroscopy interrogates the chemical environment of intracerebral lesions: elevated choline is associated
with high cellular turnover as found in neoplasms.Tumours are often centred upon white matter with relative
sparing of the overlying cortex. Gyriform enhancement is unusual in neoplasms unless there is meningeal disease.
In the acute stage, an infarct will demonstrate restricted diffusion: high signal on the DWI image and low signal
on the ADC image. Cytotoxic oedema is typical of infarcts, whilst vasogenic oedema is typical of neoplasms.
7 A patient has radiotherapy after resection of his prirpary brain tumour. There is a change in his neurological
symptoms. Which imaging feature would be more supportive of tumour recurrence rather than radiation injury?
(a) Low relative cerebral blood volume on CT perfusion imaging
(b) Decreased activity on Thallium SPECT imaging
(c) Decreased amplitude of the choline peak on MR spectroscopy
(d) High signal on MR FLAIR imaging
(e) Decreased amplitude of the N-acetyl-aspartate peak (NAA) on MR spectroscopy

7 (e)
Radiation injury Tumour recurrence
MR spectroscopy Decreased choline Decreased NAA peak
peak
CT perfusion: relativecerebral bloodvolume rCBV low Increased choline peakrCBV
(rCBV) high
Thallium SPECT Deceased activity Increased activity
8 A CT reveals an intra-axial cerebral mass.Which of the following is the most likely diagnosis?
(a) Haemangioblastoma
(b) Haemangiopericytoma
(c) Meningioma
(d) Leptomeningeal lymphoma
(e) Nasopharyngeal carcinoma

8 (b)
Haemangiopericytoma is the only intra-axial mass listed.

14
CNS Tumours
15 A 5 year old presents with symptoms of raised intracranial pressure and ataxia. A CT examination reveals
hydrocephalus and a posterior fossa mass which comprises a cyst and mural nodule.Given the mbst likely
diagnosis, which of the following isincorrect?
(a) Surrounding vasogenic oedema is common
(b) Surgical resection is the treatment of choice
(c) Intense enhancement is typical
(d) Precocious puberty is associated
(e) Disseminated disease is rare

15 (a)
A pilocytic astrocytoma is the most likely diagnosis. This is the most common paediatric glioma, and the
characteristic appearance is of acyst with an intensely enhancing mural nodule. Surrounding oedema is rare. 40%
of patients with both NF-1 and an optic pathway gllioma also suffer from precious puberty.
26 An elderly man is admitted for assessment of focal neurological symptoms. An incidental supratentorial cystic
lesion is demonstrated. Which of the following features would make the diagnosis of subdural hygroma, rather
than arachnoid cyst, more likely?
(a) Isointense to CSF on T1W MR imaging
(b) Mass effect
(c) Isointense to CSF on T2W MR imaging
(d) Flattened sulci
(e) Bony remodeling

26 (e)
Scalloping of the adjacent bone, possibly through transmitted pulsations, is often seen in arachnoid cysts. This is
never seen in subdural hygromas, but can be seen with epidermoid cysts or porencephaly.?
29 An intra-cranial cyst is seen on MR imaging. What feature would support the diagnosis of arachnoid cyst
rather than an epidermoid?
(a) Absence of enhancement
(b) Irregularity
(c) Iso-intense to CSF on DWI
(d) Cerebello-pontine angle location
(e) Calcification

29 (c)
The content of an arachnoid cyst has the same characteristics as CSF on all MR imaging sequences.
43 An elderly patient presents with confusion. A CT head reveals a lesion involving the corpus callosum and
invading both frontal lobes. There is enhancement of the lesion periphery after the administration of intravenous
contrast meaium.What is the most likely diagnosis?
(a) Abscess
(b) Radiation necrosis
(c) Meningioma
(d) Infarction
(e) Glioblastoma multiforme

43 (e)
A ring-enhancing lesion that crosses the midline may represent, amongst other conditions, a glioblastoma
multiforme ('a butterfly glioma'), astrocytoma or lymphoma. Abscesses rarely cross the midline. An infarct is less
likely as the above distribution does not correspond to a single arterial territory.
50 A CT head demonstrates a dilated lateral and third ventricle with a normal sized fourth ventricle. Which of the
following would most likely account for this appearance?
(a) Colloid cyst
(b) Pinealoma
(c) Tonsillar herniation
(d) Choroid plexus papilloma
(e) Basilar impression

50 (b)
The pattern of dilatation is consistent with obstruction at the level of the aqueduct - of the options listed, a
pinealoma is most likely to cause obstruction at this level.

15
CNS Tumours
Barret2
11 A patient presents with sensorineural hearing loss. A cerebellopontine angle mass is demonstrated on MR
imaging. Which of the following features support a diagnosis of schwannoma over menigioma?
(a) High signal intensity relative to grey matter on T2W imaging
(b) Enhancement with i. v. gadolinium
(c) Calcification
(d) Unilateral lesion
(e) Medurally based lesion

11 (a)
Schwannomas are the most common CPA mass. The epicentre of the lesion is typically in the epicenter of the
internal auditory canal, typically causing a >2 mm difference in canal diameter on the effected side. Extension
into the CPA causes an 'ice cream cone' appearance. On MR imaging, the lesion is iso-intense on T1W, hyper-
intense on T2W and has dense enhancement (however, meningiomas also typically enhance). Ca is not typical.
20 A patient presents with gradual onset of sensory symptoms affecting the right hand. CT reveals a superficial
mass within the fronto-parietal region. Which imaging feature would not support the diagnosis of menigioma?
(a) Inward bulging of the grey-white junction
(b) Surrounding oedema
(c) Hyperdense on unenhanced images
(d) Enhancement with intravenous contrast medium
(e) lntralesional haemorrhage.

20 (e)
Only 1 % of meningiomas are associated with internal haemorrhage. Regarding the other characteristics: the
inward bulging of the grey white junction is suggestive of an extra-axial lesion, such as a meningioma; 60% of
meningiomas are associated with oedema; meningiomas are typically hyperdense on unenhanced CT imaging
and are seen to enhance with i. v. contrast medium.
21 A 45 year old female presents with facial pain and abnormal eye movements. A cerebellopontine angle lesion
is identified on MR imaging. Which feature would not be consistent with an epidermoid tumour?
(a) Iso-intense to CSF on T1W Imaging
(b) Low signal on DWI
(c) Heterogeneous signal on FLAIR
(d) Lack of 1Efnhancement
(e) Low signal on balanced sequence/ true FISP

21 (b)
Epidermoid tumours are typically iso-intense to CSF on T1 and T2W imaging, with high signal on diffusion
weighted sequences and heterogeneous signal on FLAIR imaging. They do not typically enhance.
26 A female patient is seen in the endocrine clinic and a pituitary microadenoma is suspected. Regarding the
imaging of the pituitary gland, which of the following is an abnormal appearance?
(a) High signal in the posterior pituitary on T1 W
(b) Intermediate signal intensity in the posterior pituitary on T2W
(c) Enhancement of the posterior pituitary on T1 W after gadolinium administration
(d) Convex superior margin of the pituitary gland in pregnant subjects
(e) High signal in the anterior pituitary on T2W

26 (e)
On T1W imaging, the anterior pituitary is typically iso-intense and the posterior pituitary hyperintense; on T2W
imaging, both the anterior and posterior pituitary are typically iso-intense. The gland is normally flat or concave
superiorly, but may be convex in pregnancy.
29 An MRI brain is performed and a lesion is seen in the suprasellar region. Which of the following imaging
characteristics is not typical?
(a) Craniopharyngioma - calcification and nodular enhancement
(b) Epidermoid - isointense to CSF on T1 W and T2W
(c) Tuber cinerum - isointense to grey matter on T1W
(d) Schwannoma - T1W hypointensity and marked enhancement with i. v. gadolinium
(e) Rathke's cleft cyst - T1W hypointensity and lack of enhancement with i. v. gadolinium

16
CNS Tumours
29 (e)
A Rathke's cleft cyst is typically hyperintense on T1W, and has smooth peripheral enhancement with intravenous
contrast medium.

31 An adolescent undergoes MR imaging and an incidental suprasellar mass is found. What feature would suggest
a diagnosis of craniopharyngioma rather than a Rathke cleft cyst?
(a) Absence of calcification
(b) Predominantly high signal on T1 W imaging
(c) Smooth contour
(d) Homogeneous signal intensity
(e) Nodular enhancement

31 (e)
A craniopharyngioma would typically be seen as a cyst with a mural nodule. Calcification is present in 90% of
cases in the younger age group. Both craniopharyngiomas and Rathke cleft cysts have variable signal intensities
on MR imaging, but most commonly craniopharyngiomas have a low signal on T1W imaging. The enhancement
pattern is typically 'solid' or 'nodular', as opposed to the'rim-like' pattern seen in Rathke cleft cysts.

47 A 40 year old man presents with a seizure and a CT head reveals an intracranial lesion. An oligodendroglioma
is suspected. Which feature would be least expected with this diagnosis?
(a) Ill-defined enhancement
(b) Cystic degeneration
(c) Absence of oedema
(d) Calcification
(e) Hyperdense on unenhanced imaging

47 (e)
Oligodendrogliomas are slowly growing gliomas, most commonly located in the frontal lobes. Large nodular
calcification is present in approximately 90% of cases. Cystic degeneration, ill-defined enhancement and a lack
of oedema are all common. Over 80% are either hypo- or iso- dense on unenhanced CT imaging.
53 A patient has a glioma resected and the post-surgical tumour bed is examined by CT imaging. Images are
acquired both before and after the administration of i. v. contrast medium. Which of the following features would
lead one to suspect that there is residual tumour?
(a) Linear enhancement around the pre-operative tumour bed
(b) Enhancement within the pre-operative tumour bed, 1 day after surgery
(c) Enhancement within the pre-operative tumour bed, 4 days after surgery
(d) Evidence of haemorrhage
(e) Dural enhancement

53 (b)
Within the first 2 days, enhancement within the pre-operative tumourbed should be regarded as suspicious of
incomplete resection. After this time, it is common for enhancing granulation tissue to develop. Dural
enhancement is a normal post-operative finding.

54 An adolescent undergoes MR imaging after having a number of seizures. A mass is demonstrated within the
periphery of the right temporal lobe. The lesion contains a number of cysts. There is no surrounding oedema and
the mass does not enhance after the adminstration of i. v. gadolinium. Which of the following is the most likely
diagnosis?
(a) Desmoplastic infantile ganglioglioma
(b) Tanycytic ependymoma
(c) Oligodendroglioma
(d) High grade astrocytoma
(e) Dysembroplastic neuroepithelial tumour

54 (e)
DNETs are benign tumours arising from cortical gray matter, most commonly found in the temporal lobes. They
are characterised by the presence of multiple cysts and a lack of surrounding oedema. Desmoplastic infantile
gangliogliomas typically occur before the 2ndbirthday; Oligodendrogliomas typically occur in 30-60 year olds.

17
CNS Tumours
55 A patient presents with a focal neurological deficit. CT imaging reveals multiple haemorrhagic foci. Which of
the following features would be more consistent with haemorrhagic metastases, rather than haemorrhagic
cerebrovascular malformations?
(a) Incomplete haemosiderin ring
(b) Surrounding oedema within the first week from symptom onset
(c) Central enhancement
(d) Calcification
(e) Lack of enhancement with i. v. contrast medium

55 (a)
Haemorrhagic metastases typically have an incomplete or absent haemosiderin ring. Calcification is rare. Most
commonly, there is either nodular, eccentric or ring enhancement. This contrasts with haemorrhagic
cerebrovascular malformations, which typically show minimal or central enhancement. Surrounding oedema is
present in both the acute/ subacute phase.

60 A 5 year old child presents with headache, vomiting and ataxia. On CT imaging, a high attenuation posterior
fossa mass is seen to arise from the midline of the roof of the 4th ventricle. Given the most likely diagnosis, which
of the following is incorrect?
(a) There is hydrocephalus in the majority
(b) The mass tends to be hyperintense on T1 W
(c) There is an association between enhancing pial nodules and seizures
(d) In children older than 3 years, the mass tends to be resected
(e) Recurrence is common

60 (b)
The mass is most likely a medulloblastoma. These tend to be iso-to hypointense on T1W. Leptomeningeal
metastases, seen as enhancing nodules, can cause seizures. Hydrocephalus is present in 90% of cases.

62 A 50 year old patient presents with headache and visual disturbances. MR imaging is performed, and a mass
is seen within the clivus. A chordoma is suspected. Which of the following features would be unusual for this
diagnosis?
(a) Isointense to brain parenchyma on T1W
(b) Calcification
(c) Lack of tracer uptake on a 99mTc-MDP bone scintigram
(d) Lack of enhancement with i.v.gadolinium
(e) Some regions of low signal on T1 W

62 (d)
Chordomas originate from the remnants of the embryonic notochord, and may occur at any point along the neural
axis. The clivus is the second most frequently affected site. The tumour is locally aggressive,calcified in
approximately 50% (giving rise to areas of low T1W) and enhances. They are typically isointense to brain
parenchyma onT1W.

63 A patient presents with seizures. An MR of the brain reveals an apparent area of linear meningeal thickening
adjacent to an intracranial tumour.Which of the following would make the description of this thickening as a
'dural tail' less appropriate?
(a) The adjacent tumour is an extra-axial
(b) The area of thickening does not enhance
(c) The thickening is only present on two contiguous 5 mm sections
(d) The adjacent tumour is intra-axial
(e) There is adjacent hyperostosis

63 (b)
The 3 main criteria for the definition of a dural tail are: presence on at least two contiguous 5 mm sections through
the tumour (although now sections tend to be less than 5 mm thick); the greatest thickness being adjacent to the
tumour; and enhancement being greater than the tumour itself. Dural tails have classically been described in
associationwith meningiomas, but can occur with other extra-axial and even intraaxial tumours.

18
CNS Tumours
73 You are shown a set of paediatric MR and CT head images in a teaching session. There is a well defined,
lobulated mass within a dilated left lateral ventricle. There is hydrocephalus. It is hyperdense on CT and returns
moderately high signal onT2W. There is intense contrast enhancement. What is the most likely diagnosis?
(a) Colloid cyst
(b) Meningioma
(c) Chorid plexus papilloma
(d) Ependymoma
(e) Giant cell astrocytoma

(c)
The imaging characteristics are those of a choroid plexus papilloma. Meningiomas are rarely intraventricular and
most frequently occurin middle aged women. Colloid cysts occur within the third ventricle. Ependymomas
typically arise from the floor of the fourth ventricle.
Currie
1. A 20 year old male presents with the inability to gaze upwards. CT brain shows moderate hydrocephalus and
a rounded mass adjacent to the tectal plate. The mass demonstrates marked homogeneous enhancement and is not
calcified. MRI confirms a well-circumscribed, relatively homogeneous mass that is isointense to grey matter on
T2-weighted imaging. The mass is hyperintense on contrast-enhanced T1-weighted imaging. What is the most
likely diagnosis?
a. Germinoma
b. Teratoma
c. Pineoblastoma
d. Pineocytoma
e. Benign pineal cyst

1.a. Germinoma
Germinomas are germ-cell tumours arising from primordial germ cells. They frequently occur in the midline,
mostly in the pineal region but also in the suprasellar region. In men, 80% of pineal masses are germ-cell tumours,
in contrast to 50% in women. They tend to occur in children or young adults (10–25 years old). Symptoms depend
on the location but the case describes Parinaud syndrome – paralysis of upward gaze due to compression of the
mesencephalic tectum. Germinomas may also cause hydrocephalus by compression of the aqueduct of Sylvius,
thus patients may present with signs and symptoms of raised intracranial pressure. Germinomas are a known
cause of precocious puberty in children under the age of ten years. They are malignant tumours and may show
CSF seeding, making cytological diagnosis possible with lumbar puncture. They are, however, very radiosensitive
and show excellent survival rates. Pineal teratomas tend to be heterogeneous masses containing fat and
calcifications. Pineoblastoma is a highly malignant tumour which is more common in children and usually has
poor tumour margins.
2. A 40 year old female is investigated for worsening headaches. CT shows a well-defined hyperdense globular
lesion within trigone of left lateral ventricle. There is intense contrast enhancement. The most likely diagnosis is:
a. Choroid cyst
b. Ependymoma
c. Colloid cyst
d. Meningioma
e. Neurocytoma

2d. Meningioma
It is rare for meningiomas to occur intraventricularly (2–5% of all meningiomas) but they are the most common
trigonal intraventricular mass in adulthood. They tend to occur in 40 year old females.
3. A 40 year old female presents with bitemporal hemianopia. CT brain shows a large, slightly hyperdense
suprasellar lesion. The mass contains several lucent foci and there is bone erosion of the sella floor. There
is enhancement post-contrast. T1-weighted MR imaging shows a predominantly isointense mass causing
sella expansion and compression of the optic chiasm. The mass contains foci of low and high signal intensity.
What is the most likely diagnosis?
a. Craniopharyngioma
b. Meningioma
c. Rathke’s cleft cyst
d. Giant internal carotid aneurysm
e. Pituitary adenoma

19
CNS Tumours
3. e. Pituitary adenoma
Pituitary adenomas are divided into microadenomas (<1 cm) and macroadenomas (>1 cm). Macroadenomas may
present with endocrine dysfunction but are generally less active than microadenomas. Thus, macroadenomas
often present with symptoms of mass effect on the optic chiasm, or if there is lateral extension into the cavernous
sinuses patients may present with other local cranial nerve palsies (III, IV, VI). The differential diagnosis of a
suprasellar mass includes (‘SATCHMO’): Suprasellar extension of pituitary adenoma/sarcoid;
Aneurysm/arachnoid cyst; TB/teratoma (other germ-cell tumours); Craniopharyngioma; Hypothalamic glioma or
hamartoma; Meningioma/ metastases (especially breast); and Optic/chiasmatic glioma. In this case, the sellar is
widened and the floor is eroded suggesting the mass arises from the pituitary itself. Low-density/low-intensity
regions on CT/T1 MRI correspond to necrotic areas and high-signal foci on T1 MRI (found relatively frequently)
represent areas of recent haemorrhage.

5. A 35 year old male presents with ataxia and nystagmus. Blood tests reveal polycythaemia. CT head
demonstrates a mass predominantly of CSF density in the posterior fossa. Subsequent MRI shows a largely
cystic mass with an enhancing mural nodule. There is surrounding oedema but no calcification. The most
likely diagnosis is:
a. Metastasis
b. Pilocytic astrocytoma
c. Haemangioblastoma
d. Choroid cyst
e. Ependymoma

5. c. Haemangioblastoma
Haemangioblastoma is the most common primary intra-axial, infratentorial tumour in adults. They are benign
autosomal dominant tumours of vascular origin. Approximately 20% occur with von Hippel–Lindau disease.
Other associations include phaeochromocytomas, syringomyelia and spinal cord haemangioblastomas. About
20% of tumours cause polycythaemia. Typical CT and MRI appearances are of a largely cystic mass with an
enhancing mural nodule. Oedema may be absent or extensive but calcification is rare. Prognosis is 85% post-
surgical five-year survival rate. Infratentorial pilocytic astrocytomas may have very similar appearances to
haemangioblastomas but some differences exist that can help differentiate the two. Pilocytic astrocytomas
predominantly occur in children and young adults, are generally larger (>5 cm) than haemangioblastomas, may
contain calcifications and are not associated with polycythaemia.
(Ped) 6. A two year old girl presents with recurrent headaches, neck pain and vomiting. She is found to have
kyphoscoliosis and café-au-lait spots. CT brain shows a mostly cystic mass within the right cerebellar hemisphere.
There is some calcification. After contrast, there is enhancement of the cystic wall and strong enhancement of a
mural nodule. The most likely diagnosis is:
a. Haemangioblastoma
b. Medulloblastoma
c. Metastasis
d. Pilocytic astrocytoma
e. Arachnoid cyst

6. d. Pilocytic astrocytoma
Pilocytic astrocytoma is the most common paediatric glioma and accounts for approximately 85% of all cerebellar
astrocytomas in children. Peak age is between birth and nine years old. (Over 80% of haemangioblastomas occur
in adulthood.) They are associated with neurofibromatosis type 1 (café-au-lait spots and skeletal abnormalities).
The most common appearance is of a cyst with an intensely enhancing mural nodule (arachnoid cyst should be devoid of an
enhancing nodule). They occasionally calcify (calcification is rare in haemangioblastomas). They run a relatively benign
clinical course and almost never recur following surgical excision. There is no malignant transformation to anaplastic form.
(Ped) 17. A three year old boy presents with headaches and drowsiness. Examination reveals papilloedema. CT
brain shows hydrocephalus and a mildly hyperdense homogeneous mass at the trigone of the left lateral ventricle.
There is intense homogeneous enhancement post-contrast. On MRI the lesion is slightly hyperintense on T1 and
slightly hypointense on T2-weighted imaging relative to white matter. Gadolinium injection confirms an
intraventricular enhancing tumour island. The most likely diagnosis is:
a. Choroid plexus papilloma
b. Intraventricular meningioma
c. Ependymoma
d. Cavernous angioma
e. Pilocytic astrocytoma

20
CNS Tumours
17. a. Choroid plexus papilloma
Eighty-six per cent of choroid plexus papillomas occur below the age of five years and they represent
approximately 65% of choroid tumours. Large aggregation of choroid produces CSF at an abnormal rate. This
CSF overproduction contributes to hydrocephalus. The most common location in children is the trigone of the
lateral ventricle, the third ventricle is unusual and the fourth ventricle and cerebellopontine angle are more
common in adults. The tumour shows a smooth lobulated border and small calcifications are common. There is
intense homogeneous enhancement. Approximately 5% undergo malignant transformation to choroid plexus
carcinoma. Meningiomas are the most common trigonal intraventricular mass in adulthood. They rarely occur
under the age of 20. Cavernous haemangiomas tend to occur in the third to sixth decades and are located in the
subcortical cerebrum.

20. A 40 year old man undergoes investigation for seizures. Head CT with and without contrast shows a large,
round, sharply marginated, hypodense mass involving the cortex and subcortical white matter of the left frontal
lobe. The mass contains large nodular clumps of calcification. There is surrounding oedema and ill-defined
enhancement. MRI demonstrates a heterogeneous mass which is predominantly isointense to grey matter on T1
and hyperintense on T2. There is moderate enhancement. What is the most likely diagnosis?
a. Astrocytoma
b. Ganglioglioma
c. Ependymoma
d. Glioblastoma
e. Oligodendroglioma

20. e. Oligodendroglioma
This is an uncommon glioma which usually presents as a large mass at the time of diagnosis. Mean age is 30–50
years and they are more common in men than women. The majority are located in the frontal lobe (_60%),
although they can occur anywhere within the central nervous system, including the cerebellum, brainstem, spinal
cord, ventricles and optic nerve. Large nodular clumps of calcifications are present in up to approximately 90%
of tumours. Cystic degeneration and haemorrhage are uncommon. Prognosis depends on the grade of the tumour.
High-grade tumours show 20% ten-year survival whereas lowgradetumours show 46% ten-year survival.
Although astrocytomas can calcify, the calcifications are rarely large and nodular. Glioblastomas rarely calcify.
Gangliogliomas are more common in the temporal lobes and deep cerebral tissues and the majority of them (80%)
occur below the age of 30 years. Ependymomas often demonstrate fluid levels due to internal haemorrhage.

21. A 52 year old man presents following collapse. He was previously fit and well, describing only a relatively
recent history of dull back pain. Initial CT scan of the head reveals a 1.5 cm hyperdense mass at the
corticomedullary junction of the right cerebral hemisphere. The mass shows surrounding oedema which exceeds
the volume of the lesion. There is strong lesional enhancement following contrast injection. What is the most
likely diagnosis and subsequent management?
a. Glioblastoma multiforme with subsequent MRI of the brain
b. Prostatic cancer metastasis with digital rectal examination and measuring of the prostatic specific antigen
c. Acute haemorrhagic contusion with referral to the neurosurgeons for active monitoring
d. Renal cell metastasis with subsequent CT staging
e. Brain abscess with subsequent intravenous antibiotics

21. d. Renal cell metastasis with subsequent CT staging


Brain metastases account for approximately a third of all intracranial tumours and are the most common
intracranial neoplasm. They characteristically occur at the corticomedullary junction of the brain and have
surrounding oedema that typically exceeds the tumour volume. Multiple lesions are present in approximately two-
thirds of cases and should be searched for with administration of intravenous contrast. Most are hypodense on CT
unless haemorrhagic or hypercellular, hence the lesion in this case is haemorrhagic. This lends itself to a
differential of primary neoplasms which includes melanoma, renal cell carcinoma, thyroid carcinoma,
bronchogenic carcinoma and breast carcinoma. The history of back pain also suggests bone metastases.
Glioblastoma multiforme usually appears as an irregular, heterogeneous, low-density mass. Abscesses typically
demonstrate ring enhancement post-contrast and may show loculation and specules of gas. The patient’s history
describes collapse rather than headache or confusion following a fall, which moves the differential away from
traumatic contusion. Although prostate cancer typically metastasises to the vertebrae, it is an uncommon primary
site for brain metastases, especially as the lesion described is haemorrhagic.

21
CNS Tumours
23. A 30 year old female complains of increasing headaches, episodic vomiting and drowsiness. Fundoscopy
reveals papilloedema. Non-contrast CT of the head demonstrates hydrocephalus and a globular lesion within the
lateral ventricle. There are several small internal foci of calcification. MR shows the mass to be attached to the
septum pallucidum. It is isointense to grey matter on T1 and T2. It densely enhances after intravenous gadolinium.
What is the most likely diagnosis?
a. Ependymoma
b. Subependymoma
c. Central neurocytoma
d. Heterotopic grey matter
e. Meningioma

23. c. Central neurocytoma


Central neurocytoma is an intraventricular WHO grade II neuroepithelial tumour with neuronal differentiation.
This rare neoplasm tends to occur between the ages of 20–40 years. Patients typically present with symptoms and
signs of hydrocephalus. Imaging typically demonstrates a globular lesion attached to the septum pellucidum.
Calcification is considered characteristic, however it may be absent in approximately half the cases. On MRI, the
lesions are usually isointense to grey matter and show dense contrast enhancement. It is extremely uncommon to
see peritumoural oedema. Heterotopic grey matter should not enhance, contain calcium, nor be attached to the
septum pellucidum. Intraventricular meningiomas are typically located at the trigone. Subependymomas are
hyperintense to grey matter on T2 while ependymomas tend to be heterogeneous, childhood lesions that occur in
and around the fourth ventricle.

26. A 13 year old boy is investigated for chronic headache and visual disturbance. CT shows a well-defined mass
in the left middle cranial fossa. It is isodense to CSF. There are no calcifications, no surrounding oedema and no
contrast enhancement. There is erosion of the underlying calvarium. You suspect this is an arachnoid cyst but
your consultant suggests the possibility of an epidermoid cyst. What MR imaging sequence would best
differentiate the two?
a. Diffusion-weighted MR imaging
b. Gadolinium-enhanced T1-weighted imaging
c. Proton density imaging
d. MR spectroscopy
e. Perfusion-weighted MR imaging

26. a. Diffusion-weighted MR imaging


Epidermoid and arachnoid cysts can look very similar on CT and standard MRI T1- and T2-weighted imaging.
Both demonstrate signal intensity similar to CSF. Arachnoid cysts and the majority of epidermoid cysts do not
calcify nor enhance. Arachnoid cysts are typically found in the floor of the middle cranial fossa near the tip of the
temporal lobe (50%). Although intracranial epidermoid cysts are more commonly found at the cerebellar pontine
angle, they may be found in the middle cranial fossa. Both arachnoid and epidermoid cysts may have associated
bone erosion indicating their chronicity.
Diffusion-weighted imaging is useful at distinguishing between the two as epidermoids appear bright indicating
the marked restriction of water diffusion. Arachnoid cysts appear dark in keeping with signal from CSF.
(Ped) 30. An 18 month old boy is investigated for hyperactivity and laughing fits. MRI demonstrates a lesion
arising near the floor of the third ventricle, posterior to the pituitary infundibulum. It projects into the suprasellar
cistern. The lesion is isointense to grey matter on T1- and T2-weighted imaging and does not enhance following
gadolinium administration. The most likely diagnosis is:
a. Germinoma
b. Pituitary adenoma
c. Hypothalamic hamartoma
d. Rathke’s cleft cyst
e. Langerhans’ cell histiocytosis

30. c. Hypothalamic hamartoma


Hypothalamic hamartoma is not a true tumour by definition. It presents either with precocious puberty or with
gelastic seizures (paroxysms of inappropriate emotional outbursts, usually laughing).
The lesion is well defined, arising from the floor of the third ventricle/around the tuber cinereum of the thalamus
and extending inferiorly into the suprasellar cistern or interpeduncular cistern. The imaging characteristics
described are typical. They do not enhance.

22
CNS Tumours
(Ped) 32. A three year old boy presents with seizures and headaches. CT head shows a hypoattenuating mass
lying superior to the lateral ventricles, within the frontal region. It displays negative Hounsfield units and
peripheral calcification but does not enhance. There is partial agenesis of the corpus callosum. MRI of the brain
demonstrates a pericallosal tumour which is hyperintense on T1 and less hyperintense on T2-weighted imaging.
What is the most likely diagnosis?
a. Dermoid tumour
b. Lipoma
c. Teratoma
d. Neurocytoma
e. Ependymoma

32. b. Lipoma
This is a congenital tumour that results from abnormal differentiation of the meninx primitiva – that which
eventually differentiates into pia, arachnoid and internal dura mater.
They account for less than 1% of brain tumours but are associated with congenital abnormalities, most commonly
dysgenesis of the corpus callosum to some degree. This is particularly likely when the lipoma is located anteriorly
rather than posteriorly.
On CT they are well-circumscribed masses with negative Hounsfield units and occasional calcification, and they
do not enhance. Characteristically, they are T1 hyperintense and slightly less hyperintense on T2.
Dermoids and teratomas can show similar characteristics, with fat and calcium content.
Teratomas may enhance, although dermoids do not. However, the lesion is much more likely to be a lipoma given
its position (dermoids tend to be extra-axial (spinal canal); teratomas are much more commonly found around the
pineal region, floor of the third ventricle, posterior fossa and spine) and given the association with corpus callosum
abnormalities.

36. A 30 year old man presents with an orbital frontal headache and visual disturbance. CT head shows a large
mass arising from the region of the spheno-occiput and extending into the pontine cistern and towards the
hypothalamus. The mass contains amorphous calcification and is seen to cause bone destruction. There is no
reactive bone sclerosis. On MRI the mass exhibits mixed heterogeneous signal and a soapbubble appearance. The
solid components show marked contrast enhancement. What is the most likely diagnosis?
a. Meningioma
b. Metastasis
c. Chordoma
d. Plasmacytoma
e. Sphenoid sinus cyst

36. c. Chordoma
Chordomas originate from malignant transformation of notochordal cells. They are typically located in the sacrum
(50%), clivus (35%) and vertebrae (15%). They may rarely be found in the mandible, maxilla and scapula.
Spheno-occipital chordomas typically affect males and females in equal incidence and the average age ranges
from 20 to 40 years. The majority of them demonstrate bone destruction and amorphous calcification. The solid
components show variable but often marked contrast enhancement and MRI may show a ‘soap-bubble’
appearance. Bone sclerosis is rare.
The description given is that of a malignant process. Meningiomas typically affect older females and only rarely
(<1%) arise from the clivus. Plasmacytomas tend to occur in an older age group, are more common in the
thoracic/lumbar spine and are often osteolytic and grossly expansile lesions. Metastasis is a possibility, although
from the description given – particularly the calcification and amount of tumour extension – chordoma remains
the most likely diagnosis.

44. A 49 year old woman with AIDS presents with increasing headache. T1-weighted MR imaging demonstrates
a hypointense lesion in the periventricular white matter of the left parietal lobe. The lesion is hypointense on
FLAIR sequencing and is seen to cross the splenium of the corpus callosum. There is peripheral enhancement
post-contrast injection. The most likely diagnosis is:
a. Toxoplasmosis
b. Progressive multifocal leukoencephalopathy
c. Primary CNS lymphoma
d. Cryptococcosis
e. Tuberculosis

23
CNS Tumours
44. c. Primary CNS lymphoma (PCNSL)
PCNSL is the second most common cause of a CNS mass in patients with AIDS (behind toxoplasmosis). Typical
features include periventricular location with subependymal spread and crossing of the corpus callosum. Non-
contrast CT may show a hyperdense lesion due to dense cellularity (for this reason it may also be hypointense on
T2/FLAIR). There is often a paucity of oedema and frequent ring enhancement due to central necrosis (note this
is in contrast with the solid homogeneous enhancement seen with lymphoma in the immunocompetent patient).
Cryptococcosis is the most common cause of fungal infection in AIDS patients. CT is frequently normal and MRI
shows low T1 and high T2 signal intensities without enhancement in the lenticulostriate region.
Progressive multifocal leukoencephalopathy typically shows bilateral white matter lesions in the periventricular
region, centrum semiovale or subcortical white matter, which are hypointense on T1 and hyperintense on
T2/FLAIR. There is typically no oedema nor mass effect and no contrast enhancement.

(Ped) 45. A six year old boy is investigated for refractory complex partial seizures. CT demonstrates a well-
defined, hypodense lesion located in the cortex of the temporal lobe. There is underlying bone remodelling but
no calcification. On MRI the lesion demonstrates high signal on T2 and predominantly low signal on T1-weighted
imaging. There is no surrounding oedema, minimal mass effect and no contrast enhancement. The most likely
diagnosis is:
a. Glioblastoma multiforme
b. Dysembryoblastic neuroepithelial tumour (DNET)
c. Primitive neuroectodermal tumour (PNET)
d. Cavernous haemangiomas
e. Ependymoma

45. b. Dysembryoblastic neuroepithelial tumour (DNET)


DNETs are benign tumours of neuroepithelial origin which arise from the cortical/deep grey matter. They are
preferentially located supratentorially (temporal 62%, frontal 31%). CT demonstrates a hypoattenuating mass and
there may be thinning and remodelling of the underlying inner table reflecting the slow growth of the tumour.
On MRI they are hypointense on T1, hyperintense on T2 and small intratumoural cysts may be present to cause
a characteristic ‘bubbly’ appearance. There is minimal mass effect and no associated vasogenic oedema. A third
of lesions show calcification and most tumours do not enhance. If present, the enhancement is faint and patchy.
Gangliogliomas and cavernous haemangiomas are other tumours which may cause epilepsy in children.
Cavernous haemangiomas are typically dense on CT and commonly calcify. PNETs have a tendency for necrosis,
cyst formation and calcification. They also tend to be hyperdense on CT due to high nuclear to cytoplasmic ratio.

50. A 36 year old female presents following a tonic-clonic seizure. Over the preceding months she had suffered
with progressive, severe headaches. Contrast-enhanced CT brain shows lateral displacement of the internal
capsules by enlarged thalami but no abnormal enhancement. T2-weighted MRI demonstrates a diffuse,
contiguous area of hyperintensity involving the thalami, caudate and lentiform nuclei, the splenium of the corpus
callosum and the periventricular white matter. There is only minimal mass effect. T1-weighted gadolinium
imaging shows no enhancement. What is the most likely diagnosis?
a. Multiple sclerosis
b. Gliomatosis cerebri
c. Viral encephalitis
d. Adrenoleukodystrophy
e. Vasculitis

50. b. Gliomatosis cerebri (GC)


GC is a diffusely infiltrative glioma that may be present with or without a dominant mass. It must, however,
involve two or more lobes and usually involves contiguous areas. It affects all age groups and can be of varying
histological grade. Presentation may be enigmatic as the normal cerebral architecture is usually preserved.
Alternatively, patients present with seizures, headache and personality disorders. Prognosis is poor.
MRI findings include diffuse T2 (and proton density) hyperintensity throughout the white matter that usually
extends to involve the deep grey nuclei with enlargement of cerebral structures. It is often bilateral and symmetric
with minor mass effect and absence of necrosis.
The differential diagnosis of symmetric white matter lesions includes microvascular change, encephalitis,
demyelinating disease, vasculitis and leukoencephalopathy. GC is the most likely diagnosis in this scenario as
there is involvement of the corpus callosum and the pattern is infiltrative with enlargement of the thalami (cerebral
structures).

24
CNS Tumours
54. A ten year old boy undergoes investigation for recurring morning headaches and visual disturbance. On
examination he is noted to be short for his age. CT head shows a complex, partially cystic, strongly calcified
inhomogeneous suprasellar mass. On MRI the mass is seen to fill the third ventricle and cause cranial deviation
of the fornix. The mass is mostly hyperintense on T1 and markedly hyperintense on T2-weighted imaging. The
solid components enhance heterogeneously. What is the most likely diagnosis?
a. Germinoma
b. Pituitary adenoma
c. Supratentorial primitive neuroectodermal tumour (PNET)
d. Craniopharyngioma
e. Chiasmatic glioma

54. d. Craniopharyngioma
Craniopharyngiomas (CPs) are the most common sellar/suprasellar region mass in children. They are benign
lesions that originate from epithelial remnants (Rathke pouch) of the adenohypophysis and show a bimodal age
distribution with peaks at the first and second decades (75%) and in the fifth decade (25%). They are more
common in males. If the tumour presses on the pituitary gland, the patient may present with features of diabetes
insipidus. Similarly, compression on the hypothalamus may lead to growth disturbance, compression on the optic
chiasm may cause bitemporal hemianopia, and compression of ventricular outflow may cause raised intracranial
pressure from resulting hydrocephalus. Classically, CPs appear as calcified, mixed cystic and solid tumours with
enhancement of the solid component. They may be T1-bright due to proteinaceous components. Long-term
survival in children is good (>90%). Germinomas are typically non-cystic and non-calcified. PNETs frequently
show haemorrhage, necrosis and calcification and may resemble the appearance described for CPs. However,
they are rarely found in the suprasellar region and are much more common in the posterior fossa. Chiasmatic
gliomas are usually iso/hypointense on T1 and imaging usually defines optic nerve involvement.

55. A four year old girl presents with nausea, vomiting and ataxia. CT shows a hyperdense mass in the region of
the fourth ventricle. On T2-weighted MR imaging the mass is predominantly hypointense and contains areas of
both low and high signal intensity. Contrast-enhanced T1-weighted imaging demonstrates a heterogeneously
enhancing well-delineated mass that expands the fourth ventricle and causes elevation of the superior medullary
velum. There is a moderate amount of surrounding oedema. What is the most likely diagnosis?
a. Medulloblastoma
b. Ependymoma
c. Pilocytic astrocytoma
d. Metastasis
e. Subependymoma

55. a. Medulloblastoma
Medulloblastomas are highly malignant lesions that account for 30–40% of all posterior fossa tumours. They
typically arise from the roof of the fourth ventricle and 75% of cases occur in the first decade of life.
On imaging, they are typically seen as hyperdense midline vermian masses which abut the roof of the fourth
ventricle and cause hydrocephalus. There is usually mild to moderate perilesional oedema. Cystic change (high
signal on T2), haemorrhage and calcification (low signal on T2) are frequently seen.
They are fast-growing tumours and approximately 20% of cases demonstrate CSF dissemination at the time of
diagnosis. For this reason it is important to actively search for evidence of further cranial or spinal disease.
Treatment is usually a combination of surgery and radiotherapy.
Subependymomas typically occur in middle-aged or elderly patients. Pilocytic astrocytomas are typically cystic
with an enhancing nodule. Metastases are generally multiple and occur in older patients. Ependymomas are
usually hypo/isodense on CT.

25
CNS Tumours
Grainger
QUESTION 6
A 30-year-old man has a CT head to investigate headaches. This shows a low attenuation mass in the left
temporoparietal region which has similar density to CSF and shows no enhancement following contrast
administration. Which one of the following radiological findings would support a diagnosis of epidermoid cyst
rather than arachnoid cyst?
A High signal on diffusion-weighted MRI
B High signal on FLAIR MRI
C Low signal on diffusion-weighted MRI
D Thinning of the overlying bone
E Well-defined margins

ANSWER: A
Epidermoid cysts appear bright on diffusion-weighted imaging due to markedly restricted water diffusion. There
is free diffusion of water within arachnoid cysts; therefore, diffusion weighted imaging is very useful to
distinguish between the two.
QUESTION 20
A 33-year-old man undergoes an MRI brain to investigate worsening headaches, which he gets predominantly in
the mornings. Findings include a lobulated mass in the fourth ventricle, which extends via the foramen of
Magendie into the cisterna magna. The mass is predominantly low signal on Tlw images and high on T2w images.
Mild heterogeneous enhancement is seen postcontrast. What is the most likely diagnosis?
A Choroid plexus carcinoma
B Choroid plexus papilloma
C Ependymoma
D Meningioma
E Myxopapillary ependymoma

ANSWER: C
Ependymomas are usually intraventricular. The myxopapillary type tends to occur in the region of the filum
terminale in young adults.
(Ped) QUESTION 27
An 8-month-old child who was previously well presents with vomiting and an altered conscious level. A CT head
reveals significant hydrocephalus with a hyperdense mass. An MRI is arranged and reveals a lobulated mass
adjacent to the trigone of a lateral ventricle. This lesion yields low signal on both Tlw and T2w sequences with
avid enhancement postcontrast. Which one of the following is the most likely diagnosis?
A Choroid plexus tumour
B Craniopharyngioma
C Ependymoma
D Lymphoma
E Meningiomav

ANSWER: A
The age of the child and the description of the tumour fit best with a choroid plexus tumour. Another diagnosis
that should be considered is medulloblastoma, but this tends to occur in an older age group with peak presentation
at 7 years.
QUESTION 30
An 18-year-old man presents with increasing headaches. He has a CT brain which shows a solid hyperdense mass
in the posterior aspect of the third ventricle, displaying avid enhancement postcontrast and associated with
obstructive hydrocephalus. On MRI, the mass is hypointense on T2w images compared with grey matter and
contains small cystic areas. There is also a smaller lesion with similar signal characteristics in the suprasellar
region.The solid components of both lesions appear homogeneous and enhanceavidly postcontrast. What is the
most likely diagnosis?
A Colloid cyst
B Germinoma
C Pineoblastoma
D Pineocytoma
E Teratoma

26
CNS Tumours
ANSWER: B
''Pineal and suprasellar germ cell tumours may be synchronous and if so are pathognomonic. Teratomas tend to
be more heterogeneous.
(Ped) QUESTION 33
A 5-year-old child presents with vomiting, lethargy and a persistent headache. A CT head is performed and shows
a hyperdense midline posterior fossa mass, abutting the fourth ventricle with associated hydrocephalus. There is
significant peritumoral oedema but no calcification, and avid homogeneous enhancement is seen postcontrast. On
MRI, the mass yields low signal on Tlw and is isointense on T2w sequences. Which one of the following posterior
fossa tumours is the most likely diagnosis?
A Cerebellar haemangioblastoma
B Choroid plexus tumour
C Ependymoma
D Medulloblastoma
E Pilocytic astrocytoma

ANSWER: D
Medulloblastoma is the most common posterior fossa tumour in children, accounting for 30-^10%.
QUESTION 34
A 50-year-old man has a CT head after sustaining a head injury during a mechanical fall. The only positive finding
is a large low attenuation lesion in the left middle cranial fossa which is well defined and of the same attenuation
as CSF. There is some thinning of the overlying temporal bone.Which one of the following is the most likely
diagnosis?
A Arachnoid cyst
B Cerebral infarct
C Colloid cyst
D Dermoid cyst
E Epidermoid cyst

ANSWER: A
Arachnoid cysts are commonly an incidental finding. They do not calcify or enhance and they show identical
imaging characteristics to CSF.
QUESTION 41
A 24-year-old man is referred to the neurology outpatient clinic with worsening headaches. Neurological
examination reveals nystagmus and ataxia. A CT head shows a solitary, large, cystic mass in the posterior fossa
with a solid, enhancing component. What is the most likely diagnosis?
A Colloid cyst
B Glioblastoma multiforme
C Haemangioblastoma
D Pilocytic astrocytoma
E Pleomorphic xanthoastrocytoma

ANSWER: C
Haemangioblastoma is the commonest primary intra-axial tumour below the tentorium cerebelli in adults.
Pilocytic astrocytomas have a similar appearance but are rare in adults.
QUESTION 43
A 54-year-old man presents with headaches which are worse in the mornings. On examination, he has a
bitemporal hemianopia. MRI reveals apredominantly solid suprasellar mass, which contains a small cystic
component of high signal on Tlw and T2w images. Intense enhancement of the solid component is seen following
administration of intravenous gadolinium.Which is the most likely diagnosis?
A Craniopharyngioma
B Medulloblastoma
C Pituitary macroadenoma
D Pituitary microadenoma
E Rathke's cleft cyst

ANSWER: A
The main differential diagnosis would be a Rathke's cleft cyst but this would not show enhancement or
calcification (calcification is more common inchildhood craniophaiyngiomas).

27
CNS Tumours
QUESTION 44 A 41-year-old woman has a 5-month history of left-sided tinnitus. She undergoes an MRI scan
which reveals a tumour arising from the left cerebellopontine angle and extending into the internal auditory
meatus. It has mixed solid and cystic components. What is the most likely diagnosis?
A Epidermoid cyst
B Glomus tympanicum
C Haemangioma
D Meningioma
E Vestibular schwannoma

ANSWER: E
QUESTION 55
A 32-year-old woman presents with infertility and irregular periods. Investigations performed by her GP
demonstrate a markedly elevated prolactin level. What would be the most likely MRI finding?
A A 6-mm avidly enhancing mass in the posterior pituitary
B A 6-mm poorly enhancing mass in the anterior pituitary
C A 6-mm poorly enhancing mass in the posterior pituitary
D A 20-mm avidly enhancing mass in the anterior pituitary
E A 20-mm avidly enhancing mass in the posterior pituitary

ANSWER: B
Prolactinomas are the most common functioning pituitary microadenoma.They typically arise laterally in the
anterior lobe of the pituitary gland.?
QUESTION 56
A 65-year-old man undergoes a CT head as part of a dementia screen. This shows a well-defined high attenuation
mass peripherally in the right parietal lobe. It has a broad base along the parietal bone which appears thickened.
There is uniform enhancement of the mass postcontrast. What is the likely diagnosis?
A Arachnoid cyst
B Colloid cyst
C Middle cerebral artery aneurysm
D Meningioma
E Metastasis

ANSWER: D
QUESTION 63
A 48-year-old woman presents to her GP with frontal headaches and is found to have a right 6th nerve palsy. An
MRI brain reveals a large mass at the skull base with destruction of the clivus and invasion of the right
cavernoussinus. The mass is of mixed signal on both Tlw and T2w images andthere is irregular enhancement of
the mass postcontrast. Which one of thefollowing is the most likely diagnosis?
A Chordoma
B Ependymoma
C Glomus jugulate
D Meningioma
E Plasmacytoma

ANSWER: A
The commonest site for chordomas is the spheno-occipital synchondrosis ofthe clivus.
QUESTION 72 A 4-year-old boy is admitted via the GP with a 2-week history of headaches, nausea and
vomiting. He is ataxic and the suspected clinical diagnosis is medulloblastoma. Which one of the following
radiological findings wouldsupport this diagnosis?
A Hyperdense cerebellar mass in the midline with heterogeneous enhancement
B Hyperdense mass in the pons with homogeneous enhancement
C Isodense left cerebellar mass with a small cystic component
D Large non-enhancing midline cerebellar cyst
E Mixed cystic and solid suprasellar mass with calcification

ANSWER: A
Medulloblastoma is typically a hyperdense midline vermian mass with perilesional oedema, patchy enhancement
and hydrocephalus.

28
CNS Tumours
Oxford
1. A 9-year-old male presents to the paediatric A&E department with a history of increasing drowsiness
over the last 24 hours. Neurological assessment reveals that there is absence of upward gaze. A CT brain
is requested which reveals hydrocephalus, with marked dilatation of the lateral and third ventricles. The
fourth ventricle is unremarkable and there is obliteration of the ambient cistern due to mass effect from a
hyperdense mass noted posterior to the third ventricle. This mass has some central areas of calcification.
There is no evidence of haemorrhage. An MRI is carried out following insertion of a shunt to decompress
the ventricles. On sagittal sequences a lesion is located between the splenium of the corpus callosum and
the tectal plate, which exerts mass effect. This lesion is of intermediate signal intensity on both T1WI and
T2WI, and displays avid contrast enhancement. Enhancing meningeal lesions are also noted in the spinal
cord, indicating seeding. What is the most likely diagnosis?
A. Pituitary teratoma.
B. Meningioma.
C. Pineoblastoma.
D. Germinoma.
E. Pineal cyst.

1. D. Germinoma.
The lesion is described in the pineal region, so the differential of pineal tumours should beconsidered. These are
differentiated between germ cell tumours and pineal cell tumours. Germinomas account for 40% of all pineal
region masses and are much more frequent in males than females. They are also the most common germ cell
tumour, with teratomas and choriocarcinoma having different imaging characteristics. The main differential in
this case is between pineoblastoma and germinoma, as both occur in patients of this age group and both are
hyperdense on CT. Imaging features described to help differentiate are the avid enhancement, which is more
characteristic of germinomas, but can occur in either. Central calcification is seen commonly in germinomas, but
is uncommon in pineoblastomas, and when it occurs is often peripheral, giving the impression of an ‘exploded’
pineal gland. Subarachnoid seeding is seen in both tumours and if present CSF sampling can yield a tissue
diagnosis. Pineocytomas occur in an older age group and in frequently cause subarachnoid seeding.

10. A 50-year-old male patient is referred from A&E for a CT brain. He has a history of headache for 2
months and increasing clumsiness. The CT shows a lesion abutting the sphenoid in the anterior cranial
fossa. It measures 4 cm in size and demonstrates heterogeneous enhancement. There is noevidence of
calcification. The lesion is displacing and effacing the thir dventricle, causing mild hydrocephalus in the
lateral ventricles. An MRI is performed. The lesion is isointense on T1WI and T2WI, and demonstrates an
enhancing dural tail and broad dural base. Again the enhancement pattern is heterogeneous. A number of
flow voids are noted in the lesion.There is little peritumoural oedema noted. An MR angiogram is
performed, which indicates dual supply to the lesion from the internal and external carotid artery. What
lesion typically demonstrates these imaging findings?
A. WHO 1 meningioma.
B. WHO 3 meningioma.
C. Haemangiopericytoma.
D. Melanocytoma.
E. Schwannoma.

10. C. Haemangiopericytoma.
The imaging findings clearly describe an extra-axial tumour. These are classed as either lesions of meningeal
origin or tumours of neurogenic origin. The broad dural base and dural tail indicates that this is a tumour of
meningeal origin. Meningioma is the most common of these tumours, but the imaging characteristics are not
typical. World Health Organization (WHO) 1 and 2 meningiomas demonstrate uniform enhancement, although
haemorrhage into meningiomas is recognized. WHO 3 meningiomas (malignant meningiomas) are indicated on
imaging when there is invasion of the adjacent parenchyma, which is not described. Otherwise the diagnosis of
malignant meningioma is made by an aggressive pattern of growth on serial imaging and biopsy. The dual arterial
supply is also atypical. While meningiomas can have dual supply, they more typically derive their arterial supply
from the external carotid (via the meningeal artery). Melanocytomas are suggested on imaging by increased signal
on T1WI and are more commonly infratentorial. Without this the diagnosis is again reached most commonly
following biopsy. All the features described are typical of haemangiopericytoma.

29
CNS Tumours
3. A 45-year-old female patient is referred to you with a history of mild confusion and new onset seizures.
This patient has a history of breast cancer and is currently undergoing chemotherapy following a wide
local excision with axillary radiotherapy. The patient has a low-grade pyrexia and a low neutrophil count.
An MRI reveals a solitary lesion in the right frontal lobe, which is low/intermediate signal on T1WI and
high signal on T2WI. The lesion demonstrates ring enhancement. Diffusion-weighted imaging (DWI)
reveals slightly increased signal on B1000 imaging, with increased signal on B0 imaging, and this area is
bright on the apparent diffusion coefficient (ADC) map. What is the most likely diagnosis?
A. Abscess.
B. Metastasis.
C. Glioblastoma multiforme.
D. Infarct.
E. Radiotherapy change.

3. B. Metastasis.
The answer options give some of the classical radiological differentials for a ring-enhancing lesion in the brain:
MAGIC DR (metastasis, abscess, glioblastoma multiforme, infarct, contusion, demyelinating conditions and
post-radiotherapy change). The clinical history should steer the reader toward the first two. DWI is reasonably
useful at differentiating between these two (exceptions being in some fungal infections and toxoplasmosis), with
necrotic metastasis having unrestricted diffusion and abscess having restricted diffusion. The appearances in this
case are slightly confused by the DWI b1000 imaging showing increased signal. However, the presence of
increased signal on the ADC map correctly identifies this as being T2 shine through, showing the importance of
checking both sequences.

5. A 40-year-old man is being investigated by the neurologists for new onset epilepsy. An
electroencephalogram (EEG) indicates an epileptogenic focus in the left temporal lobe. An MRI is carried
out. The hippocampal structuresare unremarkable. A 2-cm lesion is noted in the subcortical region of the
left temporal lobe. This lesion demonstrates mild enhancement. Areas of low signal on T1WI and T2WI
are felt to represent foci of calcification. There is also an area of high signal on T1WI and T2WI seen
inferiorly within the lesion, which probably represents an area of haemorrhage. There is a rim of
surrounding oedema noted on T2WI and fluid-attenuated inversion recovery (FLAIR). What is the most
likely diagnosis?
A. Haemangioblastoma.
B. Desmoplastic infantile ganglioglioma (DIG).
C. Dysembryoplastic neuroepithelial tumour (DNET).
D. Pleomorphic xanthoastrocytoma (PXA).
E. Oligodendroglioma.

5. E. Oligodendroglioma.
Prior to evaluating the imaging characteristics, the patient’s demographics should be consideredin this case, as in
all cases of intracranial masses, as it will help limit the differential diagnosis significantly. The patient is an adult,
thus making DIG and DNET unlikely. Secondly, considerthe temporal lobe location. Haemangioblastomas are
usually infratentorial in location, especially in the absence of a history of von Hippel–Lindau syndrome. All the
other lesions are classical temporal lobe tumours. Finally, considering the imaging characteristics, both PXA and
oligodendrogliomas may be entirely solid, but cysts are commonly seen in PXAs. Calcification is seen in 60–
80% of oligodendrogliomas, but is rarely seen in PXAs. Similarly haemorrhage, while not typical of
oligodendrogliomas, is rare in PXAs.

(Ped) 12. A 5-year-old boy is admitted for investigation of headache and vomiting. Unenhanced CT
demonstrates a hyperdense mass centred on the cerebellar vermis and effacing the fourth ventricle.
Homogenous enhancement is demonstrated on contrast administration. What is the most likely diagnosis?
A. Ependymoma.
B. Pilocytic astrocytoma.
C. Haemangioblastoma.
D. Brainstem glioma.
E. Medulloblastoma.

30
CNS Tumours
12. E. Medulloblastoma.
This is the most common malignant posterior fossa tumour in children, generally occurring before 10 years of
age. The vast majority (85%) arise in the cerebellar vermis. They are hyperdense due to their high cellular content.
Calcification is uncommon, occurring in up to 20% of cases. Pilocytic astrocytoma typically present as a cystic
mass with an enhancing nodule. The most common location is the cerebellum, but they usually occur in other
sites when associated with neurofibromatosis type 1. Haemangioblastomas are rare in children and even in the
setting of von Hippel-Lindau syndrome typically manifest in early adulthood. Ependymomas are usually more
heterogenous owing to calcification, cystic change, and haemorrhage. The tumour arises from ependymal cells
that line the ventricular system and central canal of the spinal cord. Brainstem gliomas are hypodense on CT and
may show exophytic growth into the adjacent cisternal spaces.

14. An 8-year-old female patient presents to your paediatric neurology service with a history of increasing
ataxia, repeated headaches, and vomiting, increasing in severity over the last 5 months. Clinical
examination reveals marked cerebellar signs of past pointing and dysdiadochokinesis. An MRI is
requested, which shows a solid mass in the posterior fossa measuring 2cm in size. This mass arises in the
left cerebellar hemisphere and displaces the fourth ventricle. It is of low intensity on T1WI and high signal
on T2WI. There is only a small rim of surrounding oedema. The lesion demonstrates relatively
homogeneous moderate enhancement. There is no evidence of subarachnoid seeding. What is the most
likely diagnosis?
A. Pilocytic astrocytoma.
B. Ependymoma.
C. Medulloblastoma.
D. Metastasis.
E. Lhermitte–Duclos syndrome.

14. A. Pilocytic astrocytoma.


This question deals with the classical neurological differential diagnosis of a posterior fossa mass in a child. While
there are many causes, pilocytic astrocytoma and medulloblastoma account for over 60% of all childhood
posterior fossa masses. Pilocytic astrocytomas have a classical appearance of being cystic lesions with an avidly
enhancing mural nodule. However, 30% of pilocytic astrocytomas are solid tumours. In differentiating them from
medulloblastomas, pilocytic astrocytomas often arise more peripherally and displace the fourth ventricle, where
as medulloblastomas usually arise centrally from the vermis. Subarachnoid seeding is seen in up to 50% of cases
of medulloblastoma. Ependymomas are also included in the differential. As these arise from the ependyma lining
theventricle, they tend to be centred on the fourth ventricle in children. Metastases are the most common cause
of a posterior fossa mass in adults, but are less common in children.

15. A 30-year-old male patient is referred from ENT for an MRI with a historyof tinnitus and slight hearing
loss on the left side. A lesion is noted in theleft cerebellopontine angle. This extends along the nerve and
expands theinternal auditory canal. A separate nerve is noted to enter the anteriorsuperior portion of the
internal auditory canal. The lesion is isointense tothe pons on all pulse sequences. The lesion makes an
acute angle with thepetrous bone. There is no evidence of a dural tail following enhancement.What is the
most likely cause?
A. Meningioma.
B. Facial nerve schwannoma.
C. Vestibular nerve schwannoma.
D. Epidermoid.
E. Arachnoid cyst.

15. C. Vestibular nerve schwannoma.


Even in the absence of the imaging findings this would be a reasonable bet as it accountsfor 75% of CPA tumours.
However, assessment of the imaging characteristics increases thelikelihood of this diagnosis. The second most
common cause of CPA tumour is meningioma(10%). Differentiating features are lack of a dural tail, acute angle
with the petrous bone (seenin schwannomas, less commonly with meningiomas, which make an obtuse angle),
and, mostimportantly, expansion of the internal auditory canal (IAC). Facial nerve schwannomas can appearin
the same site and appear identical, but they only constitute 4% of lesions in this site and assuch would not be the
most likely cause. The facial nerve lies in the antero-superior portion ofthe IAC, although it cannot usually be
separated from the mass in the IAC on imaging.

31
CNS Tumours
17. A patient is having an MRI scan carried out to investigate a possible rightfrontal astrocytoma,
incidentally detected on CT following a head injury.The MRI features are typical of an astrocytoma, with
no evidence ofnecrosis or callosal involvement to indicate glioblastoma multiforme (GBM).MRS has been
carried out to help assess the grade of this tumour. WhatMRS features indicate a high grade lesion?
A. Elevated choline, reduced N-acetyl aspartate (NAA), choline/creatine (Cho/Cr) ratioof 1.
B. Elevated choline, reduced NAA, Cho/Cr ratio of 2.
C. Normal choline, elevated NAA.
D. Reduced choline, reduced NAA. Cho/Cr ratio of 1.2.
E. All normal, these are unaffected by tumour grade.

17. B. Elevated choline, reduced NAA, Cho/Cr ratio of 2.


NAA is thought to be a marker of neuronal integrity, choline indicates cell turnover, and creatineindicates cell
metabolism. Lactate is not detectable in normal brain spectra but is elevated in inflammation, infarction, and some
neoplasms. Most brain conditions, whether neoplastic, vascular,or demyelinating, are associated with a reduction
in NAA. A notable exception is Canavan’sdisease, which causes a rise in NAA. Choline is elevated in many
disorders, but is markedlyincreased in high-grade neoplasms. It has been reported that the ratio of choline to
creatinecan be used to help grade tumours, with a ratio over 1.5 indicating high grade in the majority ofcases. A
reduced choline and NAA in an area of tumour can indicate necrosis.

25. A 42-year-old woman has a CT scan of brain performed for the investigationof headache. A lesion with
density similar to cerebrospinal fluid is noted atthe right cerebellopontine angle. A subsequent MRI scan
is performed andthis shows that the lesion is well defined, of decreased signal on T1WI andincreased signal
on T2WI. You think the lesion is most likely an epidermoidcyst, but wish to exclude an arachnoid cyst.
Which of the following furtherMRI sequences will be most helpful in achieving this aim?
A. Proton density.
B. STIR.
C. T1 with fat suppression.
D. FLAIR.
E. T1 post gadolinium.

25. D. FLAIR.
On a FLAIR sequence, epidermoid cysts can be differentiated from arachnoid cysts because theformer show
mixed iso- to hyperintense signal, but with poor demarcation, while the signal of thelatter is suppressed, like the
signal of CSF. DWI offers a findingspecific for extra-axial epidermoid cysts by showing very high
signal.Restricted ADC compared to CSF, almost comparable to that of the brain and T2 shine-througheffect, both
play an important role in the high signal intensity of epidermoid cyst at DWI. This cantherefore be useful in
helping to distinguish them from arachnoid cyst.

53. A 45-year-old female undergoes an MRI of the pituitary that demonstrates a kidney-shaped lesion
located centrally in the pituitary fossa in the axial plane. It is hyperintense on T1WI and hypointense on
T2WI. There is no enhancement following gadolinium administration and no fluid–fluid level. What is the
diagnosis?
A. Rathke cleft cyst.
B. Craniopharyngioma.
C. Cholesterol granuloma.
D. Haemorrhagic adenoma.
E. Lipoma.

53. A. Rathke cleft cyst.


These are benign cystic lesions of the pituitary fossa derived from the Rathke pouch. They are usually
asymptomatic. Rathke cleft cysts are located in the midline between the anterior and posterior pituitary lobes and
have a characteristic kidney shape on axial images. They arehomogenously hyperintense on T1WI, due to high
protein concentration, and hypointenseon T2WI, due to low intracystic water content. They do not enhance
following contrast administration. Absence of a fluid–fluid level is helpful in differentiating Rathke cleft cyst
from hemorrhagic adenoma. Acute haemorrhage has heterogenous signal on T2WI and may demonstrate thin
peripheral enhancement on T1WI. Craniopharyngiomas have variable solid, cystic, and calcified components.
They demonstrate heterogenous enhancement following contrast. They may be intrasellar or suprasellar. A
pseudo-fluid–fluid level may occasionally be seen in craniopharyngioma. Lipoma and cholesterol granuloma are
hyperintense on both T1WI and T2WI.

32
CNS Tumours
41. A 12-year-old male with a history of gelastic seizures is referred for MRI ofthe brain. Which of the
following statements regarding hamartomas of thetuber cinereum is true?
A. No change in size, shape, or signal intensity on follow-up MRI.
B. Demonstrate homogenous contrast enhancement.
C. Calcification is a common finding.
D. Hyperintense on T1WI and T2WI, and hypointense on fat suppressed sequences.
E. Located in the sella turcica.

41. A. No change in size, shape, or signal intensity on follow-up MRI.


Hypothalamic hamartomas are developmental malformations located in the tuber cinereum ofthe hypothalamus.
The typical patient is male, in the first or second decade of life, presenting withprecocious puberty or gelastic
seizures. On MRI, they appear as well-defined pedunculated or sessile lesions that are iso/mildlyhypointense on
T1WI and iso/hyperintense on T2WI, with no contrast enhancement orcalcification. Lack of interval change
strongly supports the diagnosis.

73. A 42-year-old male is admitted with first presentation of a seizure. There is no significant past medical
history. CT demonstrates a mass lesion within the right frontal lobe. He is further investigated via MRI,
which includes MR perfusion and spectroscopy. The neurosurgical team are keen to biopsy this lesion if
there is radiological suspicion of a high-grade lesion. Which of the following radiological findings is most
consistent with a high grade lesion?
A. Relative cerebral blood volume (rCBV) 1.5 on MR perfusion.
B. Lactate peak on MRS.
C. Elevated ADC on DWI.
D. Peritumoral hyperintensity on T2WI.
E. Nodular enhancement on T1WI post gadolinium.

73. B. Lactate peak on MR spectroscopy.


Neuroepithelial tumours are either low grade (WHO I and II) or high grade (WHO III and IV).The classification
relies on pathology, but as this requires brain biopsy it is not without significant risk. Advanced MRI techniques
play an increasing role in helping to differentiate these lesions, as low-grade lesions may be managed via a ‘watch
and wait’ approach. Conventional MRI is poorly sensitive for glioma grading, as low-grade gliomas can enhance
(in up to 20%) and up to 30% of non-enhancing gliomas are malignant. MR perfusion can help differentaiate, as
rCBV tends to increase with tumour grade. A threshold of 1.75 is commonly utilized to separate low- from high
grade gliomas, but low-grade oligodendrogliomas can give misleadingly elevated rCBV values. The classic MR
spectroscopy features of high-grade lesions are elevated choline and reduced NAA.In addition an elevated lactate
signal is typical of high-grade lesions secondary to the anaerobic environment. ADC is usually lower in high-
grade lesions, but there is considerable overlap and so ADC maps are insufficient on their own for predicting
tumour grade.

74. A 54-year-old man presents with hearing loss in the left ear, which is of the sensori-neural type. He
undergoes an MRI scan of the internal auditory meati and subsequently a full MRI scan of brain. A large
extra-axial mass lesion is identified at the left cerebellopontine angle. Which of the followingfeatures on
MR imaging will be most helpful in indicating that this is probably a large vestibular schwannoma, rather
than a meningioma?
A. Enhancing dura adjacent to the mass.
B. Erosion of the adjacent porusacousticus.
C. Tumour within the adjacent internal auditory meatus.
D. Intense enhancement within the mass.
E. Hyperostosis in the adjacent petrous temporal ridge.

74. B. Erosion of the adjacent porusacousticus.


A large meningioma at the cerebellopontine angle can easily grow into the adjacent internal auditory meatus
(IAM), but typically does so without causing any enlargement or erosion of this structure. Most vestibular
schwannomas arise from the inferior vestibular nerve within the IAM and as they grow they smoothly erode the
posterior edge of the porusacousticus. Although dural enhancement has been described adjacent to vestibular
schwannomas, anenhancing ‘dural tail’ would be a finding more commonly associated with a lesion arising from
the dura, typically a meningioma. Bony hyperostosis is a finding that is associated with meningioma. Both
meningiomas andvestibular schwannomas typically show avid enhancement with intravenous gadolinium.

33
CNS Tumours
75. A 32-year-old female is referred to neurology complaining of visual disturbance and headache. She is
4 months postpartum. On examination a bitemporal hemianopia is noted. Hormonal testing reveals
hypoadrenalism and hypothyroidism. A dedicated MRI of her pituitary gland is requested. Which of the
following features is suggestive of autoimmune hypophysitis over pituitary adenoma?
A. Asymmetric pituitary enlargement.
B. Heterogenous gadolinium enhancement.
C. Loss of the posterior pituitary bright spot.
D. Sphenoid sinus mucosal thickening.
E. Age >30.

75. C. Loss of the posterior pituitary bright spot.


Autoimmune hypophysitis (AH) and non-secreting pituitary adenomas can only be differentiated with certainty
on histology. As a result, approximately 40% of patients with AH are misdiagnosedas having pituitary
macroadenoma and undergo unnecessary surgery. Hormone production is compromised in both conditions,
although a history of infertility is common with adenomas, whereas patients with AH typically achieve
spontaneous pregnancy. In an attempt to develop ascoring system to differentiate the two conditions, a recent
study found that features significantly associated with AH over adenoma were age <30, relation to
pregnancy, homogenous gadolinium enhancement, loss of the posterior pituitary bright spot, and enlarged
stalk. Features consistent with adenoma were asymmetrically enlarged pituitary, size >6 cm3, and
associated sinus mucosal thickening. The normal posterior pituitary gland is bright on T1WI due to the rich
content ofvasopressin neurosecretory granules. This is frequently lost in AH due to direct
autoimmuneinvolvement of the neurohypophysis, whereas it is conserved in the majority of adenomas, evenwhen
displaced by large tumour size.

MasterPass1
12 A 65-year-old man attended the ENT outpatients complaining of a one-year history of unilateral tinnitus and
vertigo. Audiology confirmed a unilateral sensorineural hearing loss on the affected side. An MRI scan
demonstrated a mass that was thought to be an acoustic neuroma. What signal characteristics are most likely?
a Hyperintense on T1-weighted images
b Enhancement following gadolinium
C No enhancement following gadolinium
d Hypointense on T2-weighted images
e Isointense to grey matter on T2-weighted images

12 Answer B. Enhancement following gadolinium


Acoustic neuromas are also known as vestibular schwannomas and are the most common tumours of the
cerebellopontine angle. They arise from the perineural Schwann cells of the vestibular division of the VIII cranial
nerve and are usually unilateral. In Type II neurofibromatosis they are, by definition, bilateral. A vestibular
schwannoma displays the characteristic imaging appearances that are common to all schwannomas; on CT
imaging they are isointense with brain parenchyma, exhibiting enhancement post contrast and containing cystic
components. On MRI the lesion is isointense with brain parenchyma on T1 W images, hyperintense on T2 and
displays enhancement post gadolinium.

21 A 60-year-old lady presented complaining of a hoarse voice and signs of a vocal cord palsy associated with
lower cranial nerve palsies affecting cranial nerves IX, X and XI. CT demonstrated a large extra axial soft-tissue
mass with calcification and a dural tail entering the jugular foramen. What further imaging features would you
expect?
aEn plaque spread along the skull base
b Permeative erosion of jugular foramen
C Smooth expansion
d Enhancement with contrast
e Thickening of the dura

21 Answer A: En plaque spread along the skull base


It is assumed she has a meningioma. Erosion of the jugular foramen is suggestive of a glomus jugulare tumour
while smooth expansion would be expected in a vagal schwannoma. Enhancement occurs in both neuromas and
meningiomas, and therefore is not a distinguishing feature. Thickening of the dura is seen in meningitis or dural
metastasis and is an unlikely finding in a jugular foramen meningioma.

34
CNS Tumours
47 A seven-year-old boy presented with sudden onset gait problems and subtle uncoordination on the finger nose
test. CT demonstrated a low density cystic solid lesion with subtle calcification centred on the vermis. Thick
heterogeneous enhancement was seen within the solid area along with obstructive hydrocephalus. What is
diagnosis?
a Pilocytic astrocytoma
b Medulloblastoma
c Haemangioblastoma
d Ependymoma
e Brainstem glioma

47 Answer A: Pilocytic astrocytoma


Pilocytic astrocytoma is the most likely diagnosis as it is low density on CT with calcification and nodular
enhancement. They are commonly located in the vermis (50%) and are complicated by hydrocephalus. They
commonly occur before the age of nine and are characteristically a cyst with an enhancing nodule.
Haemangioblastoma is a serious consideration, but more commonly occurs in the paravermian position; the
nodule is hyperdense on non-contrast CT and they virtually never calcify. Both lesions can be cystic with a solid
enhancing nodule. Haemangioblastomas occur more commonly in adults and as part of VHL syndrome.
48 A 62-year-old female presented to the Emergency Department with gradual onset weakness in the right upper
and lower limbs. CT demonstrated multiple rounded hyperdense lesions with marked vasogenic oedema which
showed prominent enhancement following IV contrast. Assuming the lesions are metastases, what is the most
likely primary?
a Bladder
b Renal cell carcinoma
c Colon
d Adenocarcinoma of the lung
e Uterus

48 Answer B: Renal cell carcinoma


Malignant melanoma, choriocarcinoma, oat cell, thyroid and renal cell carcinoma all cause hyperdense cerebral
metastases. They all enhance brightly with contrast.

49 A five-year-old boy was admitted with nausea and increased somnolence. Obstructive hydrocephalus was seen
on CT with a posterior fossa tumour. What typical features make medulloblastoma more likely than
ependymoma?
a Arising form the floor of the fourth ventricle
b Extension through the foramina of Luschka
C Calcification on CT
d Arising from the vermis
e Small cystic area within the mass

49 Answer D: Arising from the vermis

50 A 50-year-old female was investigated for chronic headaches. ACT demonstrated a well-defined hyperdense
parasagittal lesion with calcification that avidly enhanced with contrast. The superior sagittal sinus was invaded
and thrombus was present. What other feature on CT would you look for to help clinch the diagnosis?
a Dural tail
b Bony destruction
C Vasogenic oedema
d Ventricular distortion
e Corpus callosal involvement

50 Answer A: Dural tail


This woman has a parasagittal meningioma. Evidence of a dural tail is very typical of meningiomas but it can
also occur in dura metastasis.

35
CNS Tumours
51 A 47-year-old male was investigated for progressive left-sided weakness. On CT of his head there was loss of
grey-white matter differentiation with thickening of the cortex and minor effacement of the sulci affecting the
right frontal and temporal lobes. On MRI there was generalised high signal on T2 in this region. On FLAIR there
was diffuse enlargement of the right frontal and temporal lobes cortex with vasogenic oedema in the surrounding
white matter. What is the most likely diagnosis?
a Oligodendroglioma
b Gliomatosis cerebri
C Metastasis
d Glioblastoma multiforme
e Heterotopia

51 Answer B: Gliomatosis cerebri


Gliomatosis cerebri is a grade 3 WHO classification cerebral tumour, which affects at least two or more lobes
and is principally centred in the cortex and has little mass affect or architectural distortion. It typically affects
patient aged 40-50 years of age. On CT imaging it can be easily missed but features are loss of the grey-white
matter differentiation with minor enlargement of the cortex. On MRI imaging bright signal is seen within the
affected area and contrast is minimal. Prognosis is poor with 50% one-year survival.
52 A 25-year-old male had an MRI for chronic headaches. On MR there was a cystic mass with an enhancing
nodule in the left Sylvian fissure. The mass had an enhancing dural tail and exhibited only minor vasogenic
oedema. What is the most likely diagnosis?
a Meningioma
b Pleomorphic xanthoastrocytoma
C Anaplastic astrocytoma
d Metastasis
e Lymphoma

52 Answer B: Pleomorphic xanthoastrocytoma


Pleomorphic xanthoastrocytoma is a superficial grade 2 tumour and accounts for 1 % of all intracranial tumours.
It occurs in a younger group of patients with a mean age of 26 years. It preferentially affects the superficial
temporal lobe and is cystic with an enhancing nodule and with peritumoral vasogenic oedema.
53 A 43-year-old male underwent an MRI to evaluate hearing loss. In the right cerebellopontine angle there was
a well-defined mass which was low signal on T1W, high signal on T2W and low on FLAIR. The signal intensity
was similar to CSF There was scalloping of the bony margins. What would you expect the signal characteristics
to be on diffusion weighted imaging?
a Low signal
b Similar signal to fat
C Bright signal
d Intermediate signal
e Similar signal to the ADC map

53 Answer A: Low signal


This patient has an arachnoid cyst, which typically has similar signal characteristics to CSF (bright on T2W, low
on T1W and FLAIR). On DWI the signal characteristic is similar to CSF and hence is low
MasterPass2
14 A 40-year-old man who presented with progressive right-sided facial pain and paraesthesia underwent a CT
scan. This revealed a bibbed mass arising from Meckel's cave and ultimately a trigeminal nerve schwannoma was
diagnosed. What signal characteristics would be most characteristic on MRI?
a Isointense to grey matter on T2-weighted images
b Hyperintense on T1-weighted images
C Hypointense on T2-weighted images
d Does not enhance following IV gadolinium on T1-weighted images
e Isointense to grey matter on T1-weighted images

14 Answer E: Isointense to grey matter on T1-weighted images


A trigeminal nerve schwannoma displays the characteristic imaging appearances that are common to all
schwannomas. On CT imaging they are isointense with brain parenchyma, exhibiting enhancement post contrast
and containing cystic components. On MRI imaging the lesion is likely to be isointense with brain parenchyma
on T1, hyperintense on T2-weighted images and shows enhancement post gadolinium.

36
CNS Tumours
45 A 38-year-old immunocompromised male presented with increasing headaches and worsening memory. CT
showed three discrete hyperdense lesions in the corpus callosum and centrum semiovale. Partial enhancement
with eccentric necrotic areas was seen. What is the most likely diagnosis?
a Cryptococcosis
b Primary CNS lymphoma
C Toxoplasmosis
d Herpes encephalitis
e Tuberculosis

45 Answer B: Primary CNS lymphoma


1ry lymphoma is the most likely answer because of the position, the hyperdensity on CT and the necrotic centre.
Cerebral lymphoma is often hyperdense due to the dense cellularity, although this may not be the case in patients
who are immunocompromised. Toxoplasmosis is also a consideration but these are more often multiple. Herpes
encephalitis is an unlikely diagnosis as patient is relatively well and typical imaging features are asymmetric high
signal in the frontal and temporal lobes on T2W imaging.
46 A 32 year old had a CT head following a road traffic accident. Note was made of an incidental finding of a
low-density lesion in the right cerebellopontine angle. There was no enhancement following IV iodinated contrast.
On MRI the lesion was low on Ti and bright on T2-weighted images. DWI demonstrated signal similar to CSF
in this region with no apparent signal loss on the ADC map. What is the most likely diagnosis?
a Arachnoid cyst
b Epidermoid
C Dermoid
d Acoustic neuroma
e Choroid plexus cyst

46 Answer A: Arachnoid cyst


Arachnoid cysts are common incidental findings, which are CSF-filled cystic lesions within the arachnoid space.
They are found within the middle fossa, perisellar cisterns, retrocerebellar cisterns and cerebellopontine angle.
They are thin-walled cystic lesions which are CSF density (0-20 HU) and do not enhance with contrast. There is
often bone remodelling and compression of the underlying parenchyma. On MR imaging it has the same signal
characteristics as CSF on Ti, T2, FLAIR and DWI. They can be complicated by haemorrhage. Epidermoids appear
very similar to arachnoid cysts on CT but on FLAIR and DWI they are bright and will be brighter than CSF on
T1-weighted images.
47 A 32-year-old male presented with recent onset headaches and seizures. CT showed a rounded extra-axial
mass in the interpeduncular cistern with peripheral calcification. Low-density homogeneous foci were also seen
within the basal cisterns. No enhancement was visible post contrast. On MRI the lesion appeared bright on Ti and
T2 with a low signal rim. On proton density (PD) images it was of homogeneously high signal. What is the most
likely diagnosis?
a Lipoma
b Rathke's cleft cyst
C Epidermoid
d Arachnoid cyst
e Craniopharyngioma

47 Answer A: Lipoma
Intracranial lipoma is an uncommon mass accounting for I% of all intracranial tumours. They commonly present
in the callosal cistern but can occur in any cistern. On CT they are well defined, of similar density to fat (-100
HU) with peripheral calcification and no enhancement. On MRI the signal characteristics are similar to fat but
inhomogeneous on PD. Craniopharyngiomas are usually sellar or suprasellar. Arachnoid cysts have the same
signal as CSE Epidermoids are usually lobulated and off midline. Dermoids are usually more heterogeneous.
48 A 12-year-old male was scanned following visual complaints. An MRI showed a suprasellar lesion, which
was high signal on Ti- and T2-weighted images. Multiple areas of low signal were also seen within it and the
dorsum sellae was partially eroded. Given the patient's age and radiological features, what is likely diagnosis?
a Lipoma
b Craniopharyngioma
C Rathke's cleft cyst
d Arachnoid cyst
e Pituitary macroadenoma

37
CNS Tumours
48 Answer B: Craniopharyngioma
Craniopharyngiomas are intrasellar or suprasellar lesions and most commonly occur in the first two decades of
life. They are high signal on T2 and can be either high or low signal on T1-weighted imaging, the low signal areas
being due to calcification. Bony destruction occurs in 75% of cases and also in 75% of cases the mass is
principally cystic.

49 A six-year-old boy was investigated for precocious puberty. On sagittal T1-weighted MRI there was a
homogeneous 1-cm, well-defined soft-tissue mass between the pituitary stalk and the mammillary bodies. The
mass did not enhance with contrast. What is the most likely diagnosis?
a Craniopharyngioma
b Pituitary adenoma
C Eosinophilic granuloma
d Hypothalamic hamartoma
e Chiasmatic glioma

49 Answer D: Hypothalamic hamartoma


Hypothalamic hamartoma is a tumour of the tuber cinereum. Patients are typically less than two years of age and
present with precocious puberty and gelastic seizures (spasmodic laughter). They are most commonly seen in the
tuber cin- ereum or mammillary bodies. On CT they present as a round welldefined homogeneous mass with no
enhancement. On MRI they tend to be isointense on both Ti- and T2-weighted imaging.

50 A 13-year-old boy was being investigated for progressive confusion, lethargy and increased thirst. CT was
performed which demonstrated a well-defined, smooth-walled homogeneous hyperdense lesion in the pineal
region. At which other site does this tumour most often occur?
a Posterior fossa
b Spinal cord
C Suprasellar cistern
d Lateral ventricle
e Frontal lobe

50 Answer C: Suprasellar cistern


This patient has a germinoma and the second most common site is in the suprasellar region (2 0%). Germinomas
account for 2 % of all paediatric brain tumours and are commonly seen in males aged between 10 and 25 years
of age. Patients commonly have Parinaud's syndrome (paralysis of upward gaze). On CT there is a well-defined
hyperdense mass within the pineal gland and on MRI it is isointense on Ti and low signal on T2-weighted
imaging. On both CT and MRI there is bright enhancement with contrast.

(Ped) 51 A seven-month-old child who is crying uncontrollably and vomiting is brought to hospital. On
examination he is drowsy with a bulging fontanelle. The child undergoes a CT brain scan that shows a mildly
hyperdense mass that enhances homogeneously above the level of background brain. The mass is sitting at the
trigone of the left lateral ventricle and is large with a smooth lobulated border with no invasion. There are small
foci of calcification within the mass. The left ventricle is dilated. What is the most likely diagnosis?
a Choroid plexus carcinoma
b Choroid plexus cyst
c Choroid plexus papilloma
d Astrocytoma
e Glioblastoma multiforme

51 Answer C: Choroid plexus papilloma


Choroid plexus papillomas account for 2-5% of intracranial tumours in childhood. There is a male sex
predominance and up to 80% present before the age of one year old. They are a large collection of choroid plexus
fronds, which produce excess CSF leading to hydrocephalus. Five per cent transform into malignant choroid
plexus carcinomas, which show signs of invasion. This is an uncommon site for astrocytomas.

38
CNS Tumours
52 A 25-year-old woman had an MRI and CT for investigation of headaches. She appeared morphologically
normal and had normal blood tests. She was found to have a well-defined mass, which was homogeneously high
signal on both Ti- and T2-weighted MRI in the midline suprasellar region deviating the pituitary posteriorly. On
CT there was again a well-defined homoge- nously low-defined mass within the suprasellar region with no
enhancement. The rest of the pituitary appeared normal. What is the likely diagnosis?
a Rathke's cleft cysts
b Craniopharyngiomas
C Pituitary adenoma
d Metastasis
e Hypothalamic glioma

52 Answer A: Rathke's cleft cysts


Rathke's cleft cysts are embryological remnants of the Rathke's pouch. The cysts are lined by columnar and
cuboidal cells and occur in the intrasellar region. They are high signal on T2 and can be high or low signal on T1-
weighted imaging. Rathke's cleft cysts can compress the pituitary gland. They are usually asymptomatic but may
cause hypopituitarism. Treatment is either with cyst aspiration or cystectomy.
56 A 39-year-old man had a CT of his head, on which a fat density was seen in the most superior parts of the
frontal horns of the lateral ventricles. No other abnormalities were visible. What is the most likely reason for these
findings?
a Ruptured dermoid cyst
b Intraventricular epidermoid
C Germinal teratoma
d Intraventricular lipoma
e Metastasis

56 Answer A: Ruptured dermoid cyst


Dermoid cysts are usually midline and are well-defined masses with fat, skin, sebaceous glands and sweat glands.
They are fat density on CT and high signal on both Ti- and T2-weighted imaging. These are prone to rupture and
as the fat molecules are less dense than CSF they tend to lie in the most superior areas of the ventricles (temporal
and frontal horn of the lateral ventricles).
(Ped) 58 A three-year-old boy is taken to an optician following problems reading. The optician finds the child has
a loss of visual acuity and visual fields; he also did not think the boy looked well. He referred the child to the
hospital. The paediatrician found the boy to be thin, hyperactive and unusually alert for his age. A CT brain was
performed, which showed a mass in the suprasellar region that appeared to be extending into the optic chiasm.
This had mixed enhancement with some cystic areas and calcifications. What is the most likely diagnosis?
a Hypothalamic glioma
b Hypothalamic hamartoma
C Craniopharyngioma
d Astrocytoma
e Pituitary adenoma

58 Answer A: Hypothalamic glioma


These are the most common hypothalamic masses accounting for 10-15% of supratentorial tumours in children
and present between the ages of two and four years old. This child is showing visual deficits and diencephalic
syndrome, which is present in up to 20% of cases. The inhomogeneous enhancement is caused by tumour necrosis.
Craniopharyngioma and astrocytomas are uncommon in these sites. Hypothalamic hamartomas are rare and
usually present before the age of two years old. They are round isodense lesions that do not enhance on CT.
(Ped) 64 A three-year-old girl presents to the paediatricians with headaches and intermittent ataxia. She has an
MRI that shows a medulloblastoma. When you review the images what pattern of signal characteristics would
you expect to see in the tumour?
a Hyperintense on T1-weighted images, variable on T2-weighted images and hypointense rim post gadolinium
b Hyperintense on both Ti- and T2-weighted images without enhancement post gadolinium
C Hypointense on both Ti- and T2-weighted images, heterogeneous enhancement post gadolinium
d Hypointense on T1-weighted images, variable on T2-weighted images, homogeneous enhancement with a
hypointense rim post gadolinium
e Hypointense on T1-weighted images, hyperintense on T2-weighted images, homogeneous enhancement with a
hyperintense rim post gadolinium

39
CNS Tumours
64 Answer D. Hypointense on T1-weighted images, variable on T2-weighted images, homogeneous enhancement
with a hypointense rim post gadolinium
Medulloblastoma is the most malignant infratentorial neoplasm. The duration of symptoms is usually less than
one month prior to presentation. The most common site of tumour in this age group is the vermis cerebelli and
the roof of the fourth ventricle.

MasterPass3
23 A 40-year-old male presents to his GP with a gradual onset of increasing dizziness. Investigations in the ENT
clinic reveal a sensorineural hearing loss and gradual hemifacial spasm. Imaging studies revealed a lesion in the
cerebellopontine cistern with low Ti and high T2 signal similar to that of the CSE It was of high signal on FLAIR
sequences. What is the most likely diagnosis?
a Epidermoid
b Arachnoid cyst
C Cystic meningioma
d Acoustic neuroma
e CPA aneurysm

23 Answer A: Epidermoid
The most important differential for an epidermoid here is an arachnoid cyst. On FLAIR (fluid attenuated inversion
recovery, i.e. a `water-suppressed' pulse technique) sequences, an arachnoid cyst will be low signal similar to that
of stationary water while epidermoids are high signal. These could also be differentiated by diffusion-weighted
MR. Both an acoustic neuroma and a cystic meningioma would have areas of enhancement on T1-weighted
images.
(Ped) 39 A six-year-old child presented with loss of vision, nausea and vomiting. On examination there was
papilloedema and an ataxic gait. A brain MRI was performed, which did not show any abnormality on T1-
weighted images, but revealed a well-circumscribed hyperintense lesion with a rim of low signal, containing
multiple cysts on T2-weighted images. Which of the following is the most likely diagnosis?
a Schwannoma
b Pilocytic astrocytoma
C Primitive neuroectodermal tumour (PNET)
d Medulloblastoma
e Ependymoma

39 Answer B: Pilocytic astrocytoma


Pilocytic astrocytoma is one of the most common childhood cancers of the posterior fossa. It is also the most
benign astrocytoma and is associated with neurofibromatosis type I. The symptoms are dependent on the position
of the tumour. T2-weighted images are the differentiating sequences, as PNETs and ependymomas are isointense.
Medulloblastomas are isointense on T1-weighted images and usually have a ring of vessels between the tumour
and the brain.

47 A four-year-old boy was being investigated for short stature and diabetes insipidus. On CT there was a rounded
cystic solid mass with some peripheral calcification in the suprasellar cistern with peripheral enhancement after
administration of iodinated contrast. What is the most likely diagnosis?
a Pituitary adenoma
b Germinoma
c Optic chiasm glioma
d Craniopharyngioma
e Rathke's cleft cyst

47 Answer D: Craniopharyngioma
Craniopharyngiomas have a bimodal distribution with three-quarters occurring in the first two generations and
one-quarter within the fifth decade. Patient's symptoms depend on the site of the tumour; for example, with
diabetes insipidus when compression of the thalamus occurs or bitemporal hemianopia when compression of the
optic chiasm. Tumours occur in the tuber cinereum, suprasellar and infrasellar regions. On CT tumours appear as
a cystic solid midline mass with peripheral calcification and the solid portion enhances with contrast. On MRI
the mass is typically high signal on T1- and T2 -weighted imaging with enhancement of the solid components
with contrast.

40
CNS Tumours
48 A 35-year-old male was investigated for worsening progressive headache. On CT there was a 5 x 6-cm cystic
solid mass with heavy nodular calcification affecting the left frontal lobe. The tumour extended to the cortical
surface with associated erosion of the calvarium. There was peripheral enhancement following administration of
contrast. What is the likely diagnosis?
a Mucinous adenocarcinoma
b Oligodendroglioma
C Meningioma
d Post-radiation changes
e Glioblastoma multiforme

48 Answer B: Oligodendroglioma
Oligodendrocytoma is a slow-growing grade 2 cerebral tumour that is more common in men aged 30- 60 years.
It has a propensity for the frontal lobes and involves both the cortex and white matter. On CT there is heavy
calcification within the tumour and surrounding vasogenic oedema. Erosion of the inner table of the skull occurs,
thus aiding the differential from meningiomas. There is variable enhancement with contrast. On MRI the tumour
is hypointense on T1W and hyperintense on T2 and FLAIR except in the areas of calcification.
49 A six-year-old female child complained of a worsening headache and vomiting. On CT there was
hydrocephalus and a lobulated soft-tissue mass in the trigone of the right lateral ventricle, which enhanced brightly
with contrast. What is the cause for the hydrocephalus?
a Overproduction of CSF
b Obstruction at the foramen of Monro
C Reduced absorption
d Obstructing hydrocephalus
e Obstruction at the foramen of Luschka

49 Answer A: Overproduction of CSF


This child has a choroid plexus papilloma. They commonly occur in the lateral ventricles in children and the
fourth ventricle and CPA in adults. They are commonly smooth lobulated masses which brightly enhance with
contrast. Hydrocephalus is usually due to overproduction of CSE

50 A 45-year-old male was being investigated for reduced hearing on the left. The left internal auditory canal was
expanded by a soft-tissue mass, which extended into the left cerebellopontine angle. There was minor calcification
and the mass showed strongly enhanced following IV contrast. What is the most likely diagnosis?
a Meningioma
b Acoustic neuroma
C Epidermoid
d Metastasis
e Arachnoid cyst

50 Answer A: Meningioma
All the listed tumours occur within the cerebellopontine angle (CPA). Both meningiomas and acoustic neuromas
can cause expansion of the internal auditory canal and enhance brightly with contrast. Meningioma is the most
likely because there is calcification within the lesion described.

41
CNS Tumours
51 A 56-year-old man was being investigated for a right seventh nerve palsy. On CT there was a lobulated
heterogeneous hypodense mass extending across from the prepontine cistern to the right cerebellopontine angle
mass causing compression of the right side of the pons. The mass was slightly hyperdense relative to CSF What
is the most common signal characteristics of this mass on MRI?
a High signal on Ti and low signal on T2
b Low signal on T1 and low signal on T2
C Low signal on Ti and high signal on T2
d High signal on Ti and high signal on T2
e Intermediate signal on Ti and T2

51 Answer C: Low signal on T1 and high signal on T2


This patient most likely has an epidermoid as it is heterogeneous, lobulated and off midline. Epidermoids are
most commonly homogeneously low signal on Ti and high signal on T2-weighted imaging.
55 A 43-year-old male underwent an MRI for evaluation of hearing loss. In the right cerebellopontine angle there
was a well-defined mass, which was low signal on Ti, high signal on T2 and low signal on FLAIR with an
intensity similar to CSF There was scalloping of the bony margins. What would you expect the signal
characteristics to be on diffusion weighted imaging?
a Low signal
b Similar signal to fat
C High signal
d Intermediate signal
e Similar signal to the ADC map

55 Answer A: Low signal


This patient has an arachnoid cyst, which typically has similar signal characteristics to CSF (bright on T2W, low
on T1W and FLAIR). On DWI the signal characteristic is similar to CSF and so is low.
(Ped) 64 A seven-year-old boy presents with visual field defects. He is felt to be small for his age, taking into
account the heights of his parents. A plain skull radiograph is performed, which shows marked destruction of the
sella containing curvilinear calcifications. What is the most likely diagnosis?
a Epidermoid
b Rathke's cleft cyst
c Pituitary adenoma
d Teratoma
e Craniopharyngioma

64 Answer E: Craniopharyngioma
Craniopharyngioma can also present with diabetes insipidus from compression of the pituitary gland.
Epidermoids of the CNS usually present at a later age (10-60 years old) with different symptoms but can have
bony destruction and calcification. Rathke's cleft cysts and pituitary adenomas do not cause bony destruction and
are unlikely to contain calcification.

69 An unwell 60-year-old man who is being treated for a large diverticular abscess was witnessed to have a grand
mal seizure. He has felt generally tired recently and has a past medical history of colonic carcinoma for which he
had an extended right hemicolectomy and hemihepatectomy. An MRI scan of his brain is performed which shows
a ring enhancing lesion. What imaging features favour lesion metastasis rather than an abscess?
a Cerebellar location
b Restricted diffusion on DWI
C Uniformly thick enhancing wall
d Thinning of the medial wall of the lesion
e Vasogenic oedema

69 Answer C: Uniformly thick enhancing wall


Differentiating ring enhancing metastasis and ring enhancing abscesses can be difficult. Metastases typically
appear as thick irregular ring enhancing lesions within the corticomedullary region. Abscesses tend to
demonstrate a thinner, more uniform enhancing wall. Abscesses point towards ventricles and demonstrates
restricted diffusion.

42
CNS Tumours
Gupta
3. A 50-year-old man presents with headaches. A CT of the head reveals a 3 cm extraaxial lesion in the posterior
fossa adjacent to the tentorium. The lesion is isodense with brain on non-contrast CT and has small areas of
calcifications within. The lesion enhances homogenously with contrast. There is no surrounding oedema in the
brain parenchyma. The most likely diagnosis of the abnormality is?
(a) Medulloblastoma
(b) Meningioma
(c) Lymphoma
(d) Glioblastoma multiforme
(e) Osteosarcoma metastasis

3. (b) Meningioma
These are all typical features of a meningioma. Medulloblastoma is a childhood infratentorial tumour which is
usually hyperdense and enhances with contrast. Lymphoma can be isodense with homogenous enhancement but
usually is associated with significant peritumoral oedema and no calcifications. Glioblastoma multiforme is seen
as an irregular lesion with mass effect, oedema and a heterogenous enhancement pattern. Sarcoma metastases are
rare and would induce peritumoral oedema.
4. A 1-year-old child presents with precocious puberty and MRI shows a suprasellar mass attached to the
mamillary bodies with a thin stalk. The most likely cause is?
(a) Hypothalamic hamartoma
(b) Craniopharyngioma
(c) Hypothalamic glioma
(d) Kallman syndrome
(e) Pituitary adenoma

4. (a) Hypothalamic hamartoma


Also called hamartoma of the tuber cinereum, this is seen in children less than 2 years of age. Precocious puberty
is due to luteinising releasing hormone secretion. Craniopharyngioma presents with growth failure and visual
field defects. Kallmann syndrome presents with hypogonadism in later age. Pituitary adenomas are seen in girls
(9–13 years of age) and are usually prolactin or adrenocorticotropic hormonesecreting lesions.
20. A 50-year-old woman complains of tinnitus, headaches and hearing loss. MRI shows a heterogenous, well-
defined mass in the left cerebellopontine angle producing a local mass effect. The lesion returns low signal on
T1, heterogenous high signal T2 and heterogeneously enhances with gadolinium. The most likely diagnosis is?
(a) Schwannoma
(b) Meningioma
(c) Epidermoid
(d) Arachnoid cyst
(e) Metastasis

20. (a) Schwannoma


Vestibular schwannomas are the most common cerebellopontine angle tumours with typical imaging features as
given above. These are typically benign, slow growing tumours from Schwann cells which envelop and myelinate
cranial, spinal and peripheral nerves. In the skull, they most commonly arise in the cerebellopontine angle, from
the vestibular portion of the 8th cranial nerve. The main differential diagnosis is meningioma, which can also
grow from the 8th cranial nerve. A schwannoma is hyperintense on T2 while a meningioma is isohypointense.
Also, a meningioma forms an obtuse angle with the petrous bone while a schwannoma forms an acute angle.
Finally, a meningioma may have a dural tail, which is absent in a schwannoma.

(Ped) 5. A 10-year-old child presents to the Accident & Emergency Department with fits and visual impairment.
On MRI of the brain, a suprasellar mass was seen which returns a hyperintense signal on T1 and T2 sequences.
There is patchy enhancement with gadolinium. The most likely diagnosis of the suprasellar mass is?
(a) Germinoma
(b) Craniopharyngioma
(c) Hypothalamic hamartoma
(d) Pituitary microadenoma
(e) Suprasellar arachnoid cyst

43
CNS Tumours
5. (b) Craniopharyngioma
This has a bimodal age distribution (5-10 years and 50-60 years) presenting with fits and visual symptoms. High
signal on T1 and T2 are due to cholesterol crystals in the cyst and solid portions enhance. Germinomas are solid
tumours with homogenous enhancement. They may present with visual impairment. The lesion is hypointense on
T1 and slightly hyperintense on T2. Hypothalamic hamartoma is rare and arises from the tuber cinereum. Pituitary
microadenoma is low signal on T1. Arachnoid cysts show cerebrospinal fluid features on MRI and do not enhance.
8. A 37-year-old man presented with a history of intermittent headaches. Unenhanced CT scan of the head
demonstrates a 1 cm, dense, round lesion in the region of the interventricular foramen. Mild hydrocephalus was
also seen. On MRI, lesion returns high on T1 and T2 sequences. The most likely diagnosis of this lesion is?
(a) Meningioma
(b) Ependymoma of the 3rd ventricle
(c) Colloid cyst
(d) Dermoid cyst
(e) Arachnoid cyst
[
8. (c) Colloid cyst
Colloid cysts are seen in the region of the interventricular foramen and cause positional and intermittent
obstruction, leading to hydrocephalus and headaches. The lesion is dense on CT and high signal on T1 and T2
sequences as they commonly contain large protein molecules and the paramagnetic effect of iron and copper in
the cyst. Dermoid contains fat whereas arachnoid cysts show CSF features on imaging. Meningioma is usually
low signal on T1 and high signal on T2.
10. A 40-year-old man presents with gradually worsening symptoms of ataxia, nausea and vomiting. CT of the
head shows a 2 cm cystic lesion in the cerebellum with an enhancing mural nodule. What is the most likely
diagnosis?
(a) Cerebellar haemangioblastoma
(b) Metastasis
(c) Cystic astrocytoma
(d) Arachnoid cyst
(e) Medulloblastoma
10. (a) Cerebellar haemangioblastoma
These CT features are typical of this lesion. Metastases usually have ring-like enhancement or homogenous
enhancement rather than having a mural nodule. Cystic astrocytomas are usually > 5 cm, show calcifications,
thick walled and have no enhancing mural nodule. An arachnoid cyst is a possibility if no enhancing nodule seen.
Medulloblastoma is uncommon in adults and is usually a solid tumour with homogenous enhancement.
11. A 40-year-old teacher presents with a history of hearing loss in the left ear. Gadolinium-enhanced MRI shows
a non-enhancing lesion in the left cerebellopontine angle (CPA). The lesion is isointense to CSF on T1 and T2
sequences. On FLAIR imaging, the lesion shows incomplete attenuation of fluid signal and on diffusion-weighted
imaging it returns a bright signal.
The most likely diagnosis is?
(a) Arachnoid cyst in the left CPA
(b) Schwannoma in the left CPA
(c) Epidermoid cyst in the left CPA
(d) Lipoma in the left CPA
(e) Cystic meningioma in the left CPA
11. (c) Epidermoid cyst
This resembles CSF on non-enhanced CT. On MRI, the lesion is isointense, or slightly hyperintense to CSF.
Incomplete attenuation on FLAIR and high signal on diffusion (suggesting restricted diffusion) concludes the
diagnosis. Non enhancement is the rule.
(Ped) 20. A 6-year-old boy presents with worsening dizziness and ataxia. A CT scan of the head shows a non-
enhancing diffuse mass causing expansion of the pons and engulfing the basilar artery. On MRI, the lesion returns
low signal on T1 and high signal on T2. Post-gadolinium T1 images show no enhancement, with the tumour
involving the entire brainstem. What is the most likely diagnosis?
(a) Juvenile pilocytic astrocytoma
(b) Diffuse brainstem glioma
(c) Medulloblastoma
(d) Lymphoma
(e) Metastasis

44
CNS Tumours
20. (b) Diffuse brainstem glioma
Brainstem gliomas form up to 15% of all paediatric CNS tumours. There is an association with neurofibromatosis
type 1. They commonly present with symptoms of diplopia, weakness, unsteady gait, headache, dysarthria, nausea
and vomiting. These are poorly marginated and involve more than 50% of the brainstem at the level of maximum
involvement. Minimal or no contrast enhancement is seen.
21. A 9-year-old boy presents with precocious puberty and headache. CT of the brain shows an enhancing mass
in the pineal region with calcifications. There is moderate hydrocephalus with a dilated lateral and 3rd ventricle.
The most likely diagnosis is?
(a) Pineal germinoma
(b) Glioma
(c) Medulloblastoma
(d) Meningioma
(e) Metastases

21. (a) Pineal germinoma (also called pinealoma)


This is the most common pineal tumour and is associated with precocious puberty in children less than 10 years
old. The finding of pineal calcification before 10 years with a pineal mass enhancing with contrast is usually
diagnostic. Hydrocephalus is secondary to compression of the cerebral aqueduct. Medulloblastoma is a tumour
usually in the posterior fossa, and presenting in childhood. Suprasellar meningioma does not arise from the
pituitary fossa
(Ped) 22. A 4-year-old Caucasian child presents with loss of vision. CT of the head shows a well circumscribed
suprasellar cystic mass with rim calcifications. On MRI, the pituitary gland appears normal and the lesion has a
fluid-fluid level. The lesion returns high signal on T1, T2 and FLAIR sequences. There is minimal peripheral
enhancement with gadolinium. The most likely diagnosis is?
(a) Rathke’s cleft
(b) Epidermoid cyst
(c) Pituitary adenoma
(d) Craniopharyngioma
(e) Suprasellar arachnoid cyst

22. (d) Craniopharyngioma


Craniopharyngioma is the most common suprasellar tumour in paediatrics and usually cystic. Calcification is
seen in 90% of the cases. These lesions contain highly proteinaceous fluid, cholesterol and blood products
resulting in high signal on T1, T2 and FLAIR images.

24. A 32-year-old man with a 3 month history of headaches presents to the Accident & Emergency Department
with tonic-clonic seizures. MRI shows a 5 cm intraaxial lesion in the left frontal lobe. The lesion appears
hypointense on T1 and hyperintense on T2 to brain parenchyma. No significant surrounding oedema is seen and
there is no enhancement with gadolinium. The most likely diagnosis is?
(a) Oligodendroglioma
(b) Astrocytoma
(c) Arachnoid cyst
(d) Metastases
(e) Lymphoma

24. (b) Astrocytoma


These MRI appearances are typical of a grade II astrocytoma. Grade III are more infiltrative and show more
surrounding oedema. Oligodendrogliomas show calcifications. Arachnoid cysts show CSF density on all
sequences. Metastatic lesions and lymphoma enhance with gadolinium.

25. A 35-year-old man presents with headache and ataxia. CT of the brain shows a 6 cm cystic lesion in the right
cerebellar hemisphere with a small enhancing nodule at the margin of the cyst. The most likely diagnosis is?
(a) Arachnoid cyst
(b) Necrotic metastasis
(c) Haemangioblastoma
(d) Juvenile pilocytic astrocytoma
(e) Cysticercosis

45
CNS Tumours
25. (c) Haemangioblastoma
This is the most common posterior fossa tumour in adults after metastases. They are usually seen in the cerebellum
and there is an association with von Hippel–Lindau disease. CT appearances are typically with a large hypodense
cyst and an enhancing mural nodule. The cyst wall does not usually enhance. On MRI, flow voids may be seen
representing draining vessels adjacent to the nodule. Juvenile pilocytic astrocytoma is seen in young age and is
not associated with feeding vessels. Metastases are usually multiple in older people.
31. A 18-year-old man was admitted after a road traffic accident. CT of the head shows an incidental lesion in
the right cerebello-pontine angle. MRI shows a 4 cm homogenous lesion in the right cerebellopontine angle which
is high signal on T2, intermediate on T1 and without restricted diffusion. No gadolinium enhancement seen. What
is the most likely diagnosis?
(a) Arachnoid cyst
(b) Acoustic neuroma
(c) Epidermoid cyst
(d) Lipoma
(e) Necrotic metastasis

31. (a) Arachnoid cyst


An arachnoid cyst returns signal characteristics of cerebrospinal fluid. High on T2, intermediate on T1 and no
restriction on diffusion-weighted images.
32. A 42-year-old man presents with increasing headache and blurred vision. CT of the head shows a large lesion
in the periphery of the left parietal lobe with extensive calcification. The lesion shows heterogenous contrast
enhancement. There is a mass effect with midline shift. What is the most likely diagnosis?
(a) Ganglioglioma
(b) Calcified arteriovenous malformation
(c) Oligodendroglioma
(d) Pilocytic astrocytoma
(e) Meningioma

32. (c) Oligodendroglioma


These tumours are seen in young adults and are usually located in the peripheral cerebrum. They typically begin
in the hemispheric white matter and grow towards the cortex. They are well circumscribed but non-encapsulated.
Calcification is a common feature.
33. A 21-year-old boy with neurofibromatosis type 1 complains of visual difficulties. MRI shows abnormal
enlargement of the optic chiasm and intense and homogenous enhancement with gadolinium. The abnormality
extends into the left optic tract. What is the most likely diagnosis?
(a) Craniopharyngioma
(b) Lymphoma
(c) Neurosarcoidosis
(d) Chiasmal glioma
(e) Tuberculosis

33. (d) Chiasmal glioma


Chiasmal glioma is associated with NF1 in 15-25% cases. Craniopharyngioma tends to displace the chiasm rather
than enlarge it. It is often cystic and may show calcification. Lymphoma is more likely to involve peripheral
nerves. Tuberculosis may involve the chiasm but in the setting of basal meningitis, leptomeningeal enhancement
and multiple cranial neuropathies.
37. A 40-year-old man presents with unilateral sensorineural hearing loss. MRI shows a well-defined mass in the
left cerebellopontine angle. The lesion returns high signal on T1, T2 and FLAIR sequences. On fat-saturated T1
with contrast, the lesion returns low signal and no contrast enhancement is seen. What is the most likely diagnosis?
(a) Acoustic schwannoma with haemorrhage
(b) Lipoma
(c) Epidermoid cyst
(d) Giant aneurysm
(e) Arachnoid cyst

37. (b) Cerebellopontine angle lipoma


Lesion has characteristics of fat on all sequences (high signal on T1 and T2 with loss of signal on fat-suppression
imaging).

46
CNS Tumours
41. A 42-year-old man presents in the Accident & Emergency Department with epileptic seizure. Head CT shows
asymmetrical white matter oedema in the left parietal region with a mass effect. Post-contrast study shows a large,
irregular and peripheral enhancing lesion with a central area of low attenuation. What is the most likely diagnosis?
(a) Lymphoma
(b) Metastasis
(c) Glioblastoma multiforme
(d) Toxoplasmosis
(e) Cerebral abscess

41. (c) Glioblastoma multiforme


These tumours are typically inhomogeneous on CT and MRI, showing irregular areas of peripheral enhancement.
Tumour necrosis is a hallmark of glioblastoma multiforme.
42. A 35-year-old man presents with persistent headaches. CT of the head shows a 3 cm homogenous and
hyperdense mass with homogenous contrast enhancement. It resolved with RTx. What is most likely diagnosis?
(a) Glioma
(b) Metastases
(c) Lymphoma
(d) Sarcoidosis
(e) Oligodendroglioma

42. (c) Lymphoma


This is seen not only in immunocompromised but also immunocompetent patients. On CT, the lesion is usually
hyperdense showing homogenous enhancement. Resolution with steroids and/or RTx is a characteristic finding
of cerebral lymphoma.
43. A 42-year-old Caucasian woman presents with multiple fits. CT of the head shows multiple, small enhancing
lesions in the cortical and subcortical areas. On MRI, these lesions return low signal on T2 and hyperintense on
post-gadolinium T1. What is the most likely diagnosis?
(a) Tuberous sclerosis
(b) Calcifications
(c) Melanoma metastases
(d) Haemorrhagic metastases
(e) Lymphoma

43. (c) Melanoma metastases


The T2 shortening effect is attributed to the paramagnetic effects of iron and copper bound to melanin.
44. A 52-year-old man presents with headaches. Head CT shows a 4 cm extra-axial, homogenous, hyperdense
lesion which enhances avidly with contrast. There is hyperostosis in the adjacent part of frontal bone. What is the
most likely diagnosis?
(a) Meningioma
(b) Lymphoma
(c) Metastasis
(d) Glioma
(e) Oligodendroglioma

44. (a) Meningioma


These tumours arise from the arachnoid ‘cap’ cells of the arachnoid villi.

47
CNS Tumours
EDiR Ques
3. Which of the following statements are correct about Haemangioblastoma (HB):
(a) The most common site is the spinal cord.
(b) Is the most common primary cerebellar tumor in adults.
(c) Most patients with multiple HBs have von Hippel-Lindau disease
(d) Most HBs have homogeneously solid appearance on MRI.
(e) Most HBs do not enhance after intravenous gadolinium.

Answers:
(a) Not correct
(b) Correct
(c) Correct
(d) Not correct
(e) Not correct

Explanation:
HB is a vascular tumour of the CNS. It occurs most often in the cerebellum, where it is the most common primary
neoplasm in adults. HBs are less commonly seen in the spinal cord and rarely occur elsewhere in the CNS.
Cerebellar hemangioblastomas are traditionally classified into 4 types. Type one is a simple cyst without
macroscopic nodule. Type II (most common) is a cyst with a mural nodule. Type III: solid tumours & type IV:
solid tumours with small internal cysts.
15. Which of the following statements are correct about pineal region masses:
(a) Pineoblastomas are categorised as part of the primitive neuroectodermal tumour group.
(b) Pineoblastomas usually show poor enhancement.
(c) Germinomas are 10 times more common in males than females.
(d) Pineal germinomas are associated with Parinaud’s syndrome.
(e) Germinomas are hypodense on unenhanced CT.

Answers:
(a) Correct
(b) Not correct
(c) Correct
(d) Correct
(e) Not correct
Explanation:
Pineoblastomas show avid enhancement on post contrast images. The isodense to hyperdense on CT. They are
similar to medulloblastoma as the both are part of neuroectodermal tumour group.
16. Regarding differentiation between epidermoids and dermoids of the brain:
(a) Epidermoids more closely resemble cerebrospinal fluid on MRI.
(b) Both are formed due to enclosure of ectodermal elements when the neural tube closes.
(c) Epidermoids are more common.
(d) Epidermoids may become malignant.
(e) Fat-fluid level on imaging is highly suggestive of dermoids.

Answers:
(a) Correct
(b) Correct
(c) Correct
(d) Not correct
(e) Correct
Explanation: Dermoids and epidermoids are benign lesions and slow-growing and never become malignant.
17. Concerning cerebellopontine angle masses:
(a) Meningiomas are the second commonest cerebellopontine angle mass.
(b) Meningiomas commonly cause expansion of the internal auditory canal.
(c) Meningiomas are typically brighter on T2 weighted MRI than T1.
(d) Epidermoids have the same signal as cerebrospinal fluid on MRI.
(e) Acoustic neuromas usually enhance poorly on post-contrast scans.

48
CNS Tumours
Answers:
(a) Correct
(b) Not correct
(c) Not correct
(d) Correct
(e) Not correct

Explanation:
Meningiomas do not cause expansion of internal auditory canal. They are usually less bright on T2-weighted
MRI. Acoustic neuromas usually enhance avidly on postcontrast images.
20. Which of the following statements are correct about cerebellar medulloblastoma:
(a) Is more common in females.
(b) Is the commonest paediatric brain tumour.
(c) 75% of patents are less than 15 years of age.
(d) Calcification occurs in 40-50%.
(e) Is associated with basal cell carcinomas.

Answers:
(a) Not correct
(b) Not correct
(c) Correct
(d) Not correct
(e) Correct

Explanation:
Medulloblastoma is the second commonest paediatric tumour, second only to astrocytoma however it is the commonest
paediatric posterior fossa tumour. It occurs more commonly in males. Calcifications are seen in up to 20% of patients. Cystic
changes or necrosis are seen in up to 50%. They are usually hyperdense on CT. On MRI, they are usually hypointense to
grey matter on T1 images and variable appearance on T2-weighted images. Oedema is almost always seen.
22. Concerning intracranial lymphoma:
(a) It is usually a Hodgkin’s lymphoma.
(b) Secondary lymphoma more commonly involves the leptomeninges than the brain parenchyma.
(c) It is usually hypodense on unenhanced CT.
(d) It is normally high signal on T2 weighted images.
(e) Toxoplasmosis may mimic lymphoma in the brain.

Answers:
(a) Not correct
(b) Correct
(c) Not correct
(d) Not correct
(e) Correct

Explanation:
CNS lymphoma are usually B-cell non-Hodgkin’s lymphoma. They are usually hyperdense on unenhanced CT. There show
intermediate to low signal on T2-weighted MR images.
23. Concerning posterior fossa tumours in children:
(a) 80% of medulloblastomas arise from the vermis.
(b) Juvenile pilocytic astrocytomas are the second commonest posterior fossa tumour.
(c) Juvenile pilocytic astrocytomas usually calcify.
(d) Brainstem gliomas mostly affect the midbrain.
(e) Ependymoma seeds to the CSF in 30% of cases.

Answers:
(a) Correct
(b) Correct
(c) Not correct
(d) Not correct
(e) Correct

49
CNS Tumours
Explanation:
Juvenile pilocytic astrocytomas are at the second commonest posterior fossa tumours after medulloblastoma.
Only 20% of these calcify. Brainstem gliomas mostly affect the pons.
29. Which of the following are correct regarding dermoid and epidermoid cysts?
(a) Epidermoid cysts are usually unilocular.
(b) High signal on T1 weighted MRI is diagnostic of a dermoid cyst.
(c) Dermoid cysts may be distinguished from lipoma on T1 weighted MRI.
(d) The most common location of dermoid cyst in the head and neck is the orbit.
(e) Epidermoid cysts have high signal on T2 weighted MRI scans.

Answers:
(a) Correct
(b) Not correct
(c) Not correct
(d) Correct
(e) Correct

Explanation:
Dermoid cysts are usually hyperintense on T1-weighted MR images but this signal characteristic can be seen in
other lesions example lipoma. Hence their differentiation is not recommended on T1-weighted MR images.
Dermoid cysts are usually seen in orbit, oral and nasal cavities however orbit is the most common site.

50

You might also like